SlideShare a Scribd company logo
Welcome to
Permutations & Combinations
Table of contents
𝟎𝟑
04
20
07
Session 01
Fundamental Principle of Counting
Multiplication Principle (Rule of AND) 04
Addition Principle (Rule of OR)
Factorial Notation
𝟐𝟑
38
24
24
36
Session 02
PERMUTATION
FORMULA FOR PERMUTATION
Special Case
Permutation when all the
objects are not distinct
𝟒𝟑
44
61
Session 03
Combination
String Method
𝟔𝟓
72
81
Session 04
Complementary Principle
Including/Excluding Specified
Objects
𝟗𝟎
91
101
102
Session 05
DIVISIBILITY MODEL
Rank
Circular Permutations
𝟏𝟏𝟎
112
116
Session 06
Circular Permutations
Formation of Groups
𝟏𝟐𝟖
129
143
135
Session 07
All Possible Selections
MODEL BASED ON DIVISORS
Exponent of Prime in 𝑛!
𝟏𝟒𝟕
148
Session 08
GEOMETRY MODELS
𝟏𝟔𝟕
168
176
179
Session 09
Derangements
Principle of Inclusion and Exclusion 173
Distinct Thing → Distinct Boxes
Identical Things → Distinct Boxes
𝟏𝟖𝟕
Session 10
Application of Multinomial Theorem 191
Fundamental Principles
of Counting
Session 01
Return To Top
Key Takeaways
Total number of ways for two events simultaneously
𝑚 × 𝑛
Event 𝐴
Different ways
𝑚
Event 𝐵
Different ways
𝑛
Fundamental Principle of Counting:
Number of different ways of arranging
• Counting techniques
Selecting different objects
Multiplication Principle (Rule of AND)
Return To Top
There are 3 ways to travel from 𝐴 to 𝐵 and 4 ways to travel from
𝐵 to 𝐶. In how many ways can a person travel from 𝐴 to 𝐶 via 𝐵.
𝐴
Return To Top
𝐵 𝐶
3 ways 4 ways
Number of ways to travel from 𝐴 to 𝐵 = 3
Number of ways to travel from 𝐵 to 𝐶 = 4
Number of ways he can travel from 𝐴 to 𝐵 and 𝐵 to 𝐶 = 3 × 4 = 12 Ways
𝐵 𝐶
𝑥
𝑦
1
2
(𝑥, 1) (𝑥, 2) (𝑥, 3) (𝑥, 4)
(𝑦, 1) (𝑦, 2) (𝑦, 3) (𝑦, 4)
𝐴
(𝑧, 1) (𝑧, 2) (𝑧, 3) (𝑧, 4)
𝑧 3
4
• The Multiplication principle can be generalised for any finite number of events.
Return To Top
Key Takeaways
Addition Principle (Rule of OR):
Total number of ways for an event 𝐴 or 𝐵 to occur
𝑚 + 𝑛
Event 𝐴
Different ways
𝑚
Event 𝐵
Different ways
𝑛
Return To Top
Observe a road network in the diagram below.
𝐴
𝐵
𝐶
𝐷
In how many ways can a person travel from 𝐴 to 𝐶 via 𝐵 (or) via 𝐷.
Case I:Via 𝐵 𝐴 𝐵 𝐶
3 ways 4 ways
∴ Number of ways he can travel = 3 × 4 = 12 Ways
Case II:Via 𝐷 𝐴 𝐷 𝐶
2 ways 3 ways
∴ Number of ways he can travel = 2 × 3 = 6 Ways
Return To Top ∴ Total number of ways = 12 + 6 = 18 Ways
Key Takeaways
Addition Principle (Rule of OR):
Note:
 𝐴 and 𝐵 are disjoint.
 Can be extended to finite number of disjoint events.
Return To Top
In how many ways can we pair up one boy with one girl from a group
of 3 boys and 5 girls ?
5 Girls
3 Boys
Total possible ways = 3 × 5 = 15
15
A
3
B
5
C
8
Return To Top
D
ENTRY
EXIT
= 5 × 5
= 25
There are 5 doors to enter and exit the auditorium, but the same
door cannot be used to enter and exit the auditorium, Then in how
many ways can a student enter and exit the auditorium.
Return To Top
ENTRY
EXIT
= 5 × 4
= 20
There are 5 doors to enter and exit the auditorium, but the same
door cannot be used to enter and exit the auditorium, Then in how
many ways can a student enter and exit the auditorium.
Return To Top
4 3 5
Customer has 4 options,
Burgers Fries Beverages Deserts
2
4 × 3 × 5 × 2 = 120
Return To Top
= 5 × 4 × 3 × 2
= 120
→
→
→
→
Return To Top
= 12 × 11 × 10
= 1320
Twelve students compete for a race. The number of ways in which
first three places can be taken is ?
2𝑛𝑑
1𝑠𝑡
3𝑟𝑑
Return To Top
A customer forgets a 3-digit code for an Automated Teller Machine
(ATM) in a bank. Find the largest possible number of unsuccessful
trials necessary to obtain correct code.
Return To Top
10 × 10 × 10
= 1000
= 1000 − 1
= 999
Number of unsuccessful attempts:
Number of ways to
form 3 digit numbers
Number of ways of filling
3 vacant places in succession
Digits :1, 2, 3, 4, 5, 6
=
For 3 digit even numbers,
1 2 3 4 5 6
2 4 6
Return To Top
How many 3 digit even numbers can be formed from the digits
1,2, 3, 4,5, 6 if the digits can be repeated.
Unit’s place can be filled in 3 ways.
Ten’s place can be filled in 6 ways. (Repetition is allowed)
Hundred’s place also can be filled in 6 ways. (Repetition is allowed)
6 6 3
∴ Required number of 3-digit even number
= 6 × 6 × 3
= 108
Return To Top
How many 3 digit even numbers can be formed from the digits
1,2, 3, 4,5, 6 if the digits can be repeated.
i. Any of the 5 persons can leave the lift cabin in 7 different ways.
Thus, by product rule, the total number of ways = 7 × 7 × 7 × 7 × 7
= 75 = 16807.
7𝑡ℎ
6𝑡ℎ
5𝑡ℎ
4𝑡ℎ
3𝑟𝑑
2𝑛𝑑
1𝑠𝑡
𝐺
ii. First person can leave the lift cabin in 7 different ways.
Second person can leave the lift cabin in 6 different ways.
⋯
Thus, by product rule, the total number of ways = 7 × 6 × 5 × 4 × 3
= 2520.
5 persons entered the lift on the ground floor of an 8 −floor building (ground
floor included). Suppose each of them can leave the cabin independently at any
floor beginning with the first. In how many ways can each of the five persons can
leave the lift
i. at any of the 7 floors.
ii. at different floors.
Return To Top
Key Takeaways
Factorial Notation:
• The product of first ‘𝑛’ natural numbers are denoted by 𝑛!
𝑛! = 1 × 2 × 3 × ⋯ × 𝑛 − 1 × 𝑛
When 𝑛 = 1,1! = 1
When 𝑛 = 2, 2! = 1 × 2 = 2
When 𝑛 = 3,3! = 1 × 2 × 3 = 6
When 𝑛 = 4,4! = 1 × 2 × 3 × 4 = 24
Return To Top
NOTE:
• We define 0! = 1
• 𝑛! = 𝑛 × 𝑛 − 1 ! = 𝑛 × 𝑛 − 1 × 𝑛 − 2 ! and so on
5! = 5 × 4! = 120
6! = 6 × 5! = 720
7! = 7 × 6! = 5040
• Factorial of a negative integer is not defined.
• The value of 𝑛! ends with zero, if 𝑛 ≥ 5.
Return To Top
Key Takeaways
Find the remainder when 1! + 2! + 3! + ⋯ + 100! Is divided by 15.
(∵ 5!, 6!, ⋯ are divisible by 15)
Return To Top
Let 𝑆 = 1! + 2! + 3! + ⋯ + 100!
= 1! + 2! + 3! + 4! + 5! + ⋯ + 100!
= 1! + 2! + 3! + 4! + 15𝑘 𝑘 ∈ 𝐼
= 1 + 2 + 6 + 24 + 15𝑘
= 33 + 15𝑘
= 15 𝑘 + 2 + 3
∴ Remainder is 3.
Session 02
Permutations of distinct
and alike objects
Return To Top
PERMUTATION
• A permutation is an arrangement in a definite order of a
number of objects taken some or all at a time.
FORMULA FOR PERMUTATION
• When all the objects are distinct, the number of permutations of
𝑛 distinct objects taken ′𝑟′ at a time and the objects do not repeat is
denoted by 𝑛 𝑃
𝑟 or 𝑃 𝑛, 𝑟 .
𝑛𝑃𝑟 = 𝑛 × 𝑛 − 1 × 𝑛 − 2 × ⋯ × 𝑛 − 𝑟 + 1
Return To Top
Key Takeaways
𝑛𝑃𝑟 = 𝑛 × 𝑛 − 1 × 𝑛 − 2 × ⋯ × 𝑛 − 𝑟 + 1
Proof :
By definition, 𝑛𝑃𝑟 = number of permutations of 𝑟 things out of 𝑛 different things
= number of ways of filling up 𝑟 vacant places with 𝑛 different objects
Consider 𝑟 vacant places
1𝑠𝑡 2𝑛𝑑 3𝑟𝑑 𝑟𝑡ℎ
⋯
Return To Top
𝑛 𝑛 − 1 𝑛 − 2 𝑛 − (𝑟 − 1)
So, we are filling the vacant places in succession without repetition.
∴ 𝑛𝑃𝑟 = 𝑛 × 𝑛 − 1 × 𝑛 − 2 × ⋯ × 𝑛 − 𝑟 + 1
(By Fundamental principle of counting)
Key Takeaways
•
FORMULA FOR PERMUTATION
𝑟
𝑛𝑃 =
𝑛 !
𝑛−𝑟 !
Proof :
𝑛𝑃𝑟 = 𝑛 × 𝑛 − 1 × 𝑛 − 2 × ⋯ × 𝑛 − 𝑟 + 1
𝑟
⇒ 𝑛 𝑃 = 𝑛 × 𝑛 − 1 × 𝑛 − 2 × ⋯ × 𝑛 − 𝑟 + 1
𝑛−𝑟 !
𝑛−𝑟 !
𝑟
⇒ 𝑛𝑃 =
𝑛 !
𝑛−𝑟 !
Return To Top
Key Takeaways
𝑛 𝑃0 = 1 (Arranging nothing)
𝑛 𝑃
𝑛 = 𝑛! (Arranging all the things)
The number of permutations of 𝑛 different objects taken
𝑟 at a time,where repetition is allowed is 𝑛𝑟 .
•
•
•
Note :
Key Takeaways
Return To Top
Given: 𝑛𝑃5 = 42 ⋅ 𝑛𝑃3.
⇒
𝑛 !
= 42 ⋅
𝑛 !
𝑛−5 ! 𝑛−3 !
𝑛 − 4
⇒ 𝑛2 − 7𝑛 − 30 = 0
⇒ 𝑛 − 10 𝑛 + 3 = 0
⇒ 𝑛 = −3 (rejected)
⇒ 𝑛 = 10
= 42𝑛 𝑛 − 1 (𝑛 − 2)
Find 𝑛 such that 𝑛𝑃5 = 42 ⋅ 𝑛𝑃3.
⇒ 𝑛 𝑛 − 1 𝑛 − 2 𝑛 − 3
⇒ 𝑛 𝑛 − 1 𝑛 − 2 𝑛 − 3
⇒ 𝑛 − 3 𝑛 − 4 = 42
𝑛 − 4 − 42 = 0
(∵ 𝑛 ≥ 5)
Return To Top
Prove that:
𝑖
𝑛𝑃𝑟
𝑛𝑃𝑟−1
= 𝑛 − 𝑟 + 1
Also,𝑛 𝑃
𝑟 = 𝑛 × 𝑛 − 1 × (𝑛−2)𝑃(𝑟−2) and so on
𝑖𝑖 𝑛𝑃𝑟 = 𝑛 × 𝑛−1 𝑃 𝑟−1
𝑟
𝑖 We know, 𝑛 𝑃 =
𝑛 !
𝑛−𝑟 !
𝑛𝑃𝑟
𝑛𝑃𝑟−1
=
𝑛−𝑟 !
𝑛!
× (𝑛−𝑟+1)!
= (𝑛−𝑟+1)!
= 𝑛 − 𝑟 + 1
𝑛! 𝑛−𝑟 !
𝑛𝑃𝑟 = 𝑛 × (𝑛−1)𝑃(𝑟−1)
𝑟
𝑖𝑖 𝑛𝑃 =
𝑛−𝑟 !
𝑛!
= 𝑛 ⋅ (𝑛−1)!
𝑛−𝑟 !
Return To Top
The value of (2 ⋅ 1𝑃0 − 3 ⋅ 2𝑃1 + 4 ⋅ 3𝑃2 − ⋯ upto 51st
term) + (1! − 2! + 3! − ⋯ upto 51st term) is equal to : JEE MAINS 2020
(2 ⋅ 1𝑃0 − 3 ⋅ 2𝑃1 + 4 ⋅ 3𝑃2 − ⋯ upto 51st term) + (1! − 2! + 3! − ⋯ upto 51st term)
4 ⋅ 3𝑃2 = 4! and so on.
2 ⋅ 1𝑃0 = 2! 3 ⋅ 2𝑃1 = 3!
= 2! − 3! + 4! − ⋯ − 51! + 52! + 1! − 2! + 3! − ⋯ − 50! + 51!
= 1 + 52!
1 − 5! 5!
A
B 1 + 52!
1 + 51!
D
C
Return To Top
1
Three-digit numbers are to be formed by using the odd
digits only. The number of such numbers which satisfy the
following conditions :
I. Without repetition.
II. When repetition is allowed.
III. When at-least one digit is repeated.
Odd digits:1, 3, 5, 7, 9
I. Without repetition.
3 2!
Return To Top
Number of permutations without repetition = 5𝑃 =
5!
= 60
II. When repetition is allowed.
The number of permutations of 𝑛 different objects
taken 𝑟 at a time, where repetition is allowed is 𝑛𝑟 .
Number of permutations with repetition = 53 = 125.
125 − 60 = 65.
Return To Top
Odd digits:1, 3, 5, 7, 9
III. When at-least one digit is repeated.
Number of permutations with repetition
−
Number of permutations without repetition
Three-digit numbers are to be formed by using the odd
digits only. The number of such numbers which satisfy the
following conditions :
I. Without repetition.
II. When repetition is allowed.
III. When at-least one digit is repeated.
How many words, with or without meaning, can be made from the
letters of the word EQUATION, using each letter exactly once if :
i. 4 letters are used at a time.
ii. All letters are used but first letter is a vowel.
iii. All letters are used but last letter is a consonant.
Given word:E Q U A T I O N
8 different letters
Number of vowels = 5
Number of consonants = 3
i. 4 letters are used at a time.
Number of words = 8𝑃4 = 8 × 7 × 6 × 5 = 1680.
Return To Top
_ _ _ _ _ _ _
7!
Return To Top
How many words, with or without meaning, can be made from the
letters of the word EQUATION, using each letter exactly once if :
i. 4 letters are used at a time.
ii. All letters are used but first letter is a vowel.
iii. All letters are used but last letter is a consonant.
O
I
A Number of words = 5 × 7! = 25,200
U
E
_
5
How many words, with or without meaning, can be made from the
letters of the word EQUATION, using each letter exactly once if :
i. 4 letters are used at a time.
ii. All letters are used but first letter is a vowel.
iii. All letters are used but last letter is a consonant.
N
T
Q
_ _ _ _ _ _ _ _
3
7!
Number of words = 3 × 7! = 15,120
Key Takeaways
Special Case:
A C T :All the three letters are different.
Return To Top
• Different arrangements
A C T A T C
C A T C T A
T A C T C A
• Number of arrangements = 6 = 3! (3𝑃3)
Key Takeaways
Special Case:
3!
Return To Top
2!
Number of arrangements = = 3
•
G E G
G G E G G E
E G G :Out of three, two are identical.
E G G E G G
• Different arrangements G E G
Key Takeaways
•
Permutation when all the objects are not distinct:
The number of permutations of 𝑛 objects of which 𝑝 objects are of same
kind and rest are distinct =
𝑛 !
𝑝 !
The number of permutations of 𝑛 objects where 𝑃1objects are of one
• kind, 𝑃2 objects are of second kind, ⋯,𝑃𝑘 objects are of 𝑘𝑡ℎ kind and rest
(if any)are all distinct =
𝑛 !
Return To Top
𝑃1!⋅𝑃2!⋯𝑃𝑘!
Find the number of words that can be formed by using all the
letters of the word :
𝑖) MESOPOTAMIA 𝑖𝑖) INDEPENDENCE
Possible words =
11!
2! × 2! ×(2!)
Possible words =
12!
4! × 3! ×(2!)
Return To Top
Total letters = 11
MM AA OO ESPTI EEEE NNN DD IPC
Total letters = 12
𝑖) MESOPOTAMIA 𝑖𝑖) INDEPENDENCE
Given word :BANANA
6!
2! ×3!
= 60
6 letters
∴ Number of arrangements =
∴ Total number of words = 60
How many words can be formed using the letters of the word
BANANA ?
20
A
B 60
40
D
C
Return To Top
30
Find the number of permutations of the letters of the word
∴ Number of permutations =
9!
4! × 2!
= 7560
∴ Number of permutations = 1 ×
8!
3!×2!
= 3360
ALLAHABAD. How many of them :
𝑖) start with A. 𝑖𝑖) end with L.
𝑖𝑖𝑖) start with A and end with L.
Given word :A L L A H A B A D
A A A A L L B D H
𝑖) start with A.
A __ _ _ _ _ _ _
A A A L L B D H
Return To Top
∴ Number of permutations = 1 ×
8!
= 1680
4!
𝑖𝑖𝑖) start with A and end with L.
A __ _ _ _ _ _ L
3!
∴ Number of permutations =
7!
= 840
Find the number of permutations of the letters of the word
A A A L B D H
ALLAHABAD. How many of them :
𝑖) start with A. 𝑖𝑖) end with L.
𝑖𝑖𝑖) start with A and end with L.
𝑖𝑖) end with L.
_ _ _ _ _ _ _ _ L
A A A A L B D H
Return To Top
Combination of objects
Return To Top
Session 03
Key Takeaways
• Selection of 3 objects at a time
{A,B, D}
{A,C, D}
{B,C, D}
4 Different selections
Return To Top
Combination:
• A combination is a selection of some or all of a number of different objects where
the order of selection is immaterial.
Consider four distinct objects A B
{A,B, C}
C D
Key Takeaways
Combination:
3 elements subset of {
A, B, C, D}
Return To Top
• Number of combinations of 4 distinct objects
taken 3 at a time
= 4
{A,B, C}
{A,B, D}
{A,C, D}
{B,C, D}
In each selection, we have 3! permutations.
Number of permutations = 4 × 3! = 4𝑃3
Key Takeaways
Combination:
Number of selections of 3 objects out of 4 objects = 4
Number of arrangements of 3 objects taken all at a time = 3!
Return To Top
•
•
• Number of arrangements of 4 objects taken 3 at a time = 4 × 3! = 4𝑃3
Key Takeaways
Combination:
taken 𝑟 at a time
𝑛 𝑃𝑟
𝑟!
𝑛
= 𝐶𝑟
In general,
Return To Top
Number of selections of 𝑛 objects
× 𝑟! = 𝑛𝑃𝑟
Number of selections of 𝑛 objects
=
taken 𝑟 at a time
Key Takeaways
Combination:
• The number of combinations (selections) of 𝑛 distinct objects taken 𝑟 at a time,
𝑟
is denoted by 𝑛 𝐶 or 𝐶(𝑛, 𝑟) or
𝑛
𝑟
.
• 𝑛𝐶𝑟 𝑟 ! 𝑟 !(𝑛 −𝑟
)!
𝑛𝑃𝑟 𝑛!
= = (𝑛 ∈ 𝑁, 𝑟 ∈ 𝑊, 0 ≤ 𝑟 ≤ 𝑛)
Note
0
𝑖) 𝑛𝐶 =
𝑛!
0!𝑛 !
= 1 (Selecting nothing)
𝑛
𝑖𝑖) 𝑛𝐶 =
𝑛!
𝑛 !0!
Return To Top
= 1 (Selecting all the things)
Eight teams participate in a cricket tournament. If each team plays
once against each of the others, find the total number of matches ?
Total :8 teams
Each team plays once against
each of the others
DKR
MSK
RMI
VCB
DRH
SR
KLXI
The total number of matches = 8𝐶2 = 28
DS
Return To Top
5 boys
6 girls
5𝐶3
6𝐶4
Number of ways of selecting 3 boys = 5𝐶3 = 10
Number of ways of selecting 4 girls = 6𝐶4 = 15
∴ Total number of selections = 10 × 15 = 150
Find the number of ways of selecting 3 boys and 4 girls from
Return To Top
5 boys and 6 girls.
How many words, with or without meaning, each of 3 vowels and 2
consonants can be formed from the letters of the word INVOLUTE ?
T
L
Given word :INVOLUTE
Vowels : I O U E Consonants : N V
∴ Number of selections of 3 vowels and 2 consonants
= 4𝐶3 × 4𝐶2
For each selection, we have
5! ways
Hence, number of required ways = 4𝐶3 × 4𝐶2 × 5!
= 2880
Return To Top
Find the number of 6 - digit numbers in which all the odd digits
and only odd digits will appear.
odd digits = 1, 3, 5, 7, 9
_ _ _ _ _ _
6 digit number formed
by using 5 odd digits
⇒ 5 odd digits are to be arranged with one digit repeating in 6 places.
1 , 3 , 5 , 7 , 9 , 1
1 , 3 , 5 , 7 , 9 , 3
1 , 3 , 5 , 7 , 9 , 5
1 , 3 , 5 , 7 , 9 , 7
1 , 3 , 5 , 7 , 9 , 9
Possible selections
of digits
∴ The number of 6-digit numbers = 5𝐶1 2!
× 6!
=
Return To Top
Total number of cards = 52
13 cards of each type
Total number of black cards = 26 T
otal number of red cards
J
Q
A 2 3 4 5 6 7 8 9 10 K
A 2 3 4 5 6 7 8 9 10 K Q J
A K Q J
2 3 4 5 6 7 8 9 10
A 2 3 4 5 6 7 8 9 10 K Q J
Return To Top
J
Q
K
K Q J
K Q J
K Q J
Total number of face cards = 12
Return To Top
A 2 3 4 5 6 7 8 9 10 K Q J
SPADE
A 2 3 4 5 6 7 8 9 10 K Q J
CLUB
A 2 3 4 5 6 7 8 9 10 K Q J
DIAMOND
A 2 3 4 5 6 7 8 9 10 K Q J
HEART
Return To Top
The number of ways of choosing 4 cards from a pack of 52 playing cards,
such that :
𝑖) All are of the same suit is .
𝑖𝑖) Two are red and two are black cards is .
𝑖𝑖𝑖) All are face cards is .
Return To Top
Selection
of suit = 4𝐶1
A 2 3 4 5 6 7 8 9 10 K Q J
A 2 3 4 5 6 7 8 9 10 K Q J
A 2 3 4 5 6 7 8 9 10 K Q J
A 2 3 4 5 6 7 8 9 10 K Q J
Selection of 4 cards from same suit = 13𝐶4
𝑖) All are of the same suit = 4𝐶1 × 13𝐶4 = 2860
Return To Top
The number of ways of choosing 4 cards from a pack of 52 playing cards,
such that :
Return To Top
𝑖) All are of the same suit is .
𝑖𝑖) Two are red and two are black cards is .
𝑖𝑖𝑖) All are face cards is .
Selection of two black cards = 26𝐶2
Selection of two red cards = 26𝐶2
𝑖𝑖) Two are red and two are black cards = 26𝐶2 × 26𝐶2 = 105625
𝑖𝑖𝑖) All are face cards = 13𝐶4 = 495
Selection of one heart = 13𝐶1
Selection of one spade = 13𝐶1
One is heart and other is spade = 13𝐶1 × 13𝐶1 = 169
Find the number of ways of choosing 2 cards from a pack of 52 playing
cards, such that one is heart and other one is spade.
D
A
B
C
Return To Top
13𝐶1 × 13𝐶1
26𝐶1 × 26𝐶1
52𝐶2
13𝐶2
Given word :RANDOM
Vowels : A O
Find the number of permutations of the letters of the words RANDOM
such that vowels come together.
Consonants : R N D M
Consider as
a single unit
AO R N D M
5
∴ Number of arrangements
of 5 units = 5!
AO OA In each arrangement, two vowels can
be arranged in 2! Ways.
∴ Total number of arrangements = 5! × 2! = 240
Return To Top
The number of permutations of 𝑛 distinct things, taken all at
A time when 𝑚 specified things always come together.
𝑚! 𝑛 − 𝑚 + 1 !
Return To Top
String Method
6 women
The number of ways in which we can choose a committee from 4
men and 6 women so that the committee includes at least two men
and exactly twice as many women as men is .
4 men
Committee: At least 2 men and twice as many women as men.
Case 𝑖 : Two men and Four women
Number of selections = 4𝐶2 × 6𝐶4
Case 𝑖𝑖 : Three men and six women
Number of selections = 4𝐶3 × 6𝐶6
∴ Total number of ways = 4𝐶2 × 6𝐶4 + 4𝐶3 × 6𝐶6 = 94.
Return To Top
There are 𝑚 men and two women participating in a chess
tournament. Each participant plays two games with every other
participant. If the number of games played by the men between
themselves exceeds the number of games played between the
men and women by 84, then the value of 𝑚 is .
Given: Number of men = 𝑚,
Number of women = 2,
Number of games = 𝑚𝐶2 × 2
vs
vs
Number of games = 𝑚𝐶1 × 2𝐶1 × 2
Return To Top
2 × 𝑚𝐶2 = 2 × 𝑚𝐶1 × 2𝐶1 + 84
⇒ 2 × 𝑚 𝑚−1
2!
= 4 𝑚 + 84
⇒ 𝑚2 − 𝑚 = 4 𝑚 + 84
⇒ 𝑚2 − 5 𝑚 − 84 = 0
⇒ 𝑚 − 12 𝑚 + 7 = 0
𝑚 = −7
neglected
𝑚 = 12
There are 𝑚 men and two women participating in a chess
tournament. Each participant plays two games with every other
participant. If the number of games played by the men between
themselves exceeds the number of games played between the
men and women by 84, then the value of 𝑚 is .
Return To Top
Permutations I
ncluding
/Excluding specified Objects
Return To Top
Session 04
Given: Total number of questions = 12
Number of questions to be attempted = 8
CASE
PART (I) PART (II)
Number of
ways
Total : 5 Total : 7
𝑖 3 5 5𝐶3 × 7 𝐶5
𝑖𝑖 4 4 5𝐶4 × 7 𝐶4
𝑖𝑖𝑖 5 3 5𝐶5 × 7 𝐶3
Return To Top
In an examination, a question paper consists of 12 questions divided into
two parts ,part (𝑖) and part (𝑖𝑖), containing 5 and 7 questions, respectively.
A student is required to attempt 8 questions in all, selecting at least 3
from each part. In how many ways can a student select the questions?
∴ Total number of ways in which the student can select
= 5𝐶3 × 7𝐶5 + 5𝐶4 × 7𝐶4 + 5𝐶5 × 7𝐶3
= 210 + 175 + 35 = 420
Return To Top
In an examination, a question paper consists of 12 questions divided into
two parts ,part (𝑖) and part (𝑖𝑖), containing 5 and 7 questions, respectively.
A student is required to attempt 8 questions in all, selecting at least 3
from each part. In how many ways can a student select the questions?
The number of ways in which 5 boys and 3 girls can be arranged in a
row such that :
i) All the boys are together
.
ii) All the girls are not together
.
3 girls
5 boys
i)All the boys are together
Consider as a single unit.
5 boys sitting together,
taken as a single unit.
Return To Top
i)All the boys are together
4! ways
Return To Top
5! ways
∴ Total number of arrangements = 4! × 5! = 2880
ii)All the girls are not together
∴ Total number of arrangements in which all the girls are not together
= (Number of ways of arranging 5 boys and 3 girls)
− (Number of ways of arranging in which all the girls are together)
3 girls sitting together,
taken as a single unit
ii)All the girls are not together
(Number of ways of arranging 5 boys and 3 girls)
= 5 + 3 !
And (Number of ways of arranging in which all the girls are together)
1 + 5 = 6 units
Return To Top
3! ways
∴ Total number of arrangements = 3! × 6!
∴ Total number of arrangements in which all the girls are not together
= 5 + 3 ! − 6! × 3!
= 8! − 6! × 3! = 6! 8 × 7 − 6 = 36000
Return To Top
ii)All the girls are not together
6! ways
Complementary Principle
Note:
In the previous problem, we used complementary principle.
Number of favorable ways = Total number of ways
− Number of unfavourable ways
Return To Top
4 boys
4 girls
i)No two girls are together
5
4
So, 4 boys can be arranged in 4! ways
and 4 girls can be arranged in five places in 𝐶 × 4! ways
∴ Total number of ways = 4! × 5𝐶4 × 4! = 2880
The number of ways in which 4 boys and 4 girls can be arranged in a
row such that:
Return To Top
i) No two girls are together
.
ii)Boys and girls sit alternatively.
4 girls can be arranged in 4! ways.
4 boys can be arranged in 4! ways.
∴ Total number of ways = 4! × 4!
Hence,the total number of ways = 4! × 4! + 4! × 4! = 1152
The number of ways in which 4 boys and 4 girls can be arranged in a
row such that:
i) No two girls are together
.
ii)Boys and girls sit alternatively.
ii)Boys and girls sit alternatively
Case 1
:
4 boys can be arranged in 4! ways.
4 girls can be arranged in 4! ways
∴ Total number of ways = 4! × 4!
Case 2:
Return To Top
And consonants can be arranged in
7!
2!
∴ Total number of required arrangements = 2 × 5! ×
7!
2!
= 604800
Find the number of arrangements of the letters of the word
PERMUTATIONS such that :
Return To Top
i) All the vowels as one unit and all the consonants as one unit.
ii) There are always 4 letters between 𝑃 and 𝑆.
i) All the vowels as one unit and all the consonants as one unit.
Vowels :E U A I O Consonants: P R M T T N S
There are two units, which can be arranged in = 2! ways
In each arrangement in vowels can be arranged in = 5! ways
Remaining 10 places can be filled with the letters
10!
2!
2!
∴ Total number of ways = 7 × 2 ×
10!
= 25401600
ERMUTATION
𝑃 𝑆
𝑆 𝑃
2 cases
1+1+1+1+1+1+1 = 7
Find the number of arrangements of the letters of the word
PERMUTATIONS such that :
Return To Top
i) All the vowels as one unit and all the consonants as one unit.
ii) There are always 4 letters between 𝑃 and 𝑆.
ii)There are always 4 letters between 𝑃 and 𝑆.
∴ Total number of ways = 3! × 4!
= 144
5𝑡ℎ 6𝑡ℎ 7𝑡ℎ
1𝑠𝑡 2𝑛𝑑 3𝑟𝑑 4𝑡ℎ
3! ways
4! ways
The number of permutations of the letters of the word HEXAGON
such that
Return To Top
i)relative positions of the vowels and consonants are not changed.
ii)Order of the vowels is not changed.
Given word :HEXAGON
i) relative positions of the vowels and consonants are not changed.
Vowels : E A O Consonants : H X G N
4
7𝐶4 × 4!
Given word :HEXAGON
Vowels : E A O Consonants : H X G N
ii)Order of the vowels is not changed
Let us arrange the consonants in 4 out of 7 places.
This can be done in
Now remaining three places can be filled with vowels in
only one way. (∵ order of the vowels not to be
changed)
∴ Total number of ways = 7𝐶4 × 4! × 1 = 840
The number of permutations of the letters of the word HEXAGON
such that
Return To Top
i)relative positions of the vowels and consonants are not changed.
ii)Order of the vowels is not changed.
Note :
The number of permutations of 𝑛 distinct things, in which the order of 𝑟 things
𝑟 !
Return To Top
is not to be considered is
𝑛 !
= 𝑛 𝑃
𝑛−
𝑟
Including/Excluding Specified Objects
The number of combination of 𝑛 distinct things taken 𝑟 at a time, such that
𝑖) 𝑝 particular things are always included is
𝑖𝑖) 𝑝 particular things are always excluded is
𝑛−𝑝 𝐶 𝑟−𝑝
𝑛−𝑝 𝐶𝑟
𝑖𝑖𝑖) 𝑝 particular things are included and 𝑞 particular things are excluded is 𝑛−𝑝−𝑞 𝐶 𝑟−𝑝
Return To Top
Key Takeaways
The number of ways in which a team of 11 players can be selected
from 22 players such that
i) 2 particular players are always included is
ii) 4 particular players are always excluded is
iii)2 particular players are included and 4 particular players are
excluded is
i) 2 particular players are always included is
Given :𝑛 = 22, 𝑟 = 11
∴ 𝑛−𝑝 𝐶 𝑟−𝑝 = 20𝐶9
ii) 4 particular players are always excluded is
∴ 𝑛−𝑝 𝐶𝑟 = 18𝐶11
Return To Top
∴ 𝑛−𝑝−𝑞 𝐶 𝑟−𝑝 = 16𝐶9
Return To Top
The number of ways in which a team of 11 players can be selected
from 22 players such that
i) 2 particular players are always included is
ii) 4 particular players are always excluded is
iii)2 particular players are included and 4 particular players are
excluded is
iii) 2 particular players are always included and 4 particular players are
excluded is
∴ Number of required subsets =
Given, 𝐴 = 𝐸, 𝑄, 𝑈, 𝐴, 𝑇, 𝐼, 𝑂, 𝑁
∴ 𝑛 = 8, 𝑟 = 5
Condition 𝐴, 𝑇 are always included
8−2 𝐶 5−2
= 6𝐶3
= 20
If set 𝐴 = 𝐸, 𝑄, 𝑈,𝐴, 𝑇, 𝐼, 𝑂, 𝑁 then the number of 5 elements.
Subsets of 𝐴 which always include 𝐴, 𝑇 is
Return To Top
A guard of 12 men is formed from a group of 𝑛 soldiers. It is found
that two particular soldiers 𝐴, 𝐵 is 3 times as often together on guard
as three particular soldiers 𝐶, 𝐷, 𝐸. Then the value of 𝑛 is .
Number of soldiers = 𝑛, 𝑟 = 12
Number of ways in which 𝐴, 𝐵 are included is 3 times
that of 𝐶, 𝐷, 𝐸 included on guard.
Return To Top
C
Return To Top
E
B
A D
Number of ways in which 𝐴, 𝐵 are included is 3 times
that of 𝐶, 𝐷, 𝐸 included on guard.
Number of ways in which A and B are included = 𝑛 −2𝐶12−2
Number of ways in which C, D and E are included = 𝑛−3𝐶12−3
A guard of 12 men is formed from a group of 𝑛 soldiers. It is found
that two particular soldiers 𝐴, 𝐵 is 3 times as often together on guard
as three particular soldiers 𝐶, 𝐷, 𝐸. Then the value of 𝑛 is .
𝑛−2 𝐶 12−2
⇒ = 3 × 𝑛−3 𝐶 12−3
⇒ 𝑛−2 𝐶10 = 3 × 𝑛−3 𝐶9
⇒
𝑛−2 !
10! 𝑛−12 !
= 3 ×
𝑛−3 !
9! 𝑛−12 !
⇒ 𝑛−2
= 3
10
⇒ 𝑛 = 32
Return To Top
A guard of 12 men is formed from a group of 𝑛 soldiers. It is found
that two particular soldiers 𝐴, 𝐵 is 3 times as often together on guard
as three particular soldiers 𝐶, 𝐷, 𝐸. Then the value of 𝑛 is .
5 girls
7 boys
Consider a class of 5 girls and 7 boys. The number of different teams
consisting of 2 girls and 3 boys that can be formed from this class, if
there are two specific boys 𝐴 and 𝐵 who refuse to be the member of
the same team, is
7𝐶3
Number of ways of
selecting 2 girls = 5𝐶2
Number of ways of
selecting 3 boys =
Team consists of 2 girls and 3 boys
Total number of ways to select 2 girls and 3 boys for the team
= 5𝐶2 × 7𝐶3 = 10 × 35 = 350
Return To Top
5 girls
7 boys selecting 1 boy = 5𝐶1
Consider a class of 5 girls and 7 boys. The number of different teams
consisting of 2 girls and 3 boys that can be formed from this class, if
there are two specific boys 𝐴 and 𝐵 who refuse to be the member of
the same team, is
Number of ways of
selecting 2 girls = 5𝐶2
Number of ways of
A B
If the two boys are included, only one boy has to be chosen from 5 boys
∴ Number of ways the team can be formed if both the boys are included
= 5𝐶2 × 5𝐶1 = 10 × 5 = 50
∴ Required number of selections = (Total no. of ways) − (No. of ways the team can be
formed if both the boys are included)
= 350 − 50 = 300
Return To Top
JEE MAINS 2019
A
B
C
D
350
500
200
300
Consider a class of 5 girls and 7 boys. The number of different teams
consisting of 2 girls and 3 boys that can be formed from this class, if
there are two specific boys 𝐴 and 𝐵 who refuse to be the member of
the same team, is
Return To Top
Session 05
Rank of a word and Circular
Permutations
Return To Top
0 Multiple of 7
504,3367,5719
DIVISIBILITY MODEL:
• A number is divisible by 3: If the sum of its digits is divisible by 3.
• A number is divisible by 4: If it’s last 2 digits are divisible by 4.
• A number is divisible by 6: If it is divisible by 2 and 3.
• A number is divisible by 7: if [2 × units-place digits − (Number formed by other digits)]
Example:
Return To Top
DIVISIBILITY MODEL:
• A number is divisible by 8: If it’s last 3 digits are divisible by 8.
• A number is divisible by 9: If the sum of its digits is divisible by 9.
• A number is divisible by 11:
if (sum of the digits in the odd places) − (sum of digits in even places) is divisible by 11.
Example: 209,3564
Return To Top
Number of 5 digited numbers using the digits 0, 1, 2, 3,4 with
repetition,which are divisible by 4, is:
Given digits:0, 1, 2, 3, 4
A number is divisible by 4 if its last two digits are divisible by 4.
Last two digits can be : 00 04 12 20 24 32 40 44
8 ways
4 × 5 × 5 × 8
∴ Total number of ways = 4 × 5 × 5 × 8 = 800.
800
A
1000
B
400
C
600
Return To Top
D
216
A 600
B 240
C 3125
D
Given digits:0, 1, 2, 3, 4 and 5
A number is divisible by 3 if the sum of its digit is a multiple of 3.
3 3 + 9 = 12 9
The sum of given six digits = 0 + 1 + 2 + 3 + 4 + 5 = 15
15
⇒ Possible sum of five digits such that
the number formed is divisible by 3
= 12 or 15
Return To Top
A five-digit number divisible by 3 is to be formed using the numbers
0, 1, 2, 3, 4 and 5 without repetitions. The total number of ways this
can be done is:
Case 𝑖 : Digits are 0, 1, 2, 4, 5 (Sum of digits = 12)
Number of ways = 5𝑃5 − (Number of ways in which 0 comes in first place)
= 5! − 4!
= 120 − 24 = 96
Case 𝑖𝑖 : Digits are 1, 2, 3, 4, 5 (Sum of digits = 15)
Number of ways = 5𝑃5 = 5! = 120
∴ Total number of ways = 96 + 120 = 216
A five-digit number divisible by 3 is to be formed using the numbers
0, 1, 2, 3, 4 and 5 without repetitions. The total number of ways this
can be done is:
Return To Top
If the letters of the word MOTHER are permutated and all the words
so formed (with or without meaning) be listed as in a dictionary, then
the position of the word ‘MOTHER’ is .
Given word :MOTHER Alphabetic order of letters : E, H, M, O, R,T
From the given letters, the word MOTHER comes after the words :
1) Starting with E
2) Starting with H
3) Starting with M
𝐴) Starting with ME
𝐵) Starting with MH
𝑎) Starting with MOE
𝑏) Starting with MOH
𝐶) Starting with MO
𝑐) Starting with MOR
𝑑) Starting with MOT
𝑖) Starting with MOTE
Return To Top
Next Word :MOTHER
[JEE MAIN 2020 ]
Starting with E
E
5!
= 120
Starting with H
H
5!
= 120
Starting with ME
M
4!
= 24
E
Starting with MH M
4!
= 24
H
Starting with MOE M O E
Starting with MOH M O H
Starting with MOR M
= 6
3!
= 6
3!
= 6
3!
O R
Starting with MOTE M
2!
= 2
O T E
Return To Top
Starting with E E = 120
Starting with H
H = 120
Starting with ME M = 24
E
Starting with MH
M = 24
H
Starting with MOE
M
= 6
O E
Starting with MOH M
= 6
O H
Starting with MOR M
= 6
O R
Starting with MOTE M = 2
O E
Next word M O T H E R
T
Return To Top
∴ Position of the word
MOTHER = 309𝑡ℎ
Return To Top
Words Number of words Sum
Starting with E 120 −
Starting with H 120 240
Starting with ME 24 264
Starting with MH 24 288
Starting with MOE 6 294
Starting with MOH 6 300
Starting with MOR 6 306
Starting with MOTE 2 308
Given word :AGAIN
Alphabetic order of letters :A, A, G,I,N
Starting with A
A
Starting with G
G
4!
2!
I
= 12
4!
2!
If all the permutations of the letters of the word AGAIN are
arranged in the order as in a dictionary, then find the 49𝑡ℎ word.
Words Number of
words
Sum
Starting with 𝐴 24
Starting with 𝐺 12 36
Starting with 𝐼 12 48
49
= 12
4! = 24
The next word is
N A
Return To Top
A G I
Starting with I
Rank:
The position of a word when all permutations of that word are written in alphabetical
order is called rank.
Example:
Let the word be CAT.
All possible permutation of the word CAT: CAT, CTA, ATC, TCA, ACT, TAC.
Arranging them in alphabetical order CAT: ACT, ATC, CAT, CTA, TAC, TCA
CAT is the 3𝑟𝑑 in the above list, so the rank of the word CAT is 3.
Return To Top
Key Takeaways
Circular Permutations
The number of circular permutations of 𝑛 distinct objects is
𝑛 − 1 !
𝑎1
𝑎2
𝑎3
𝑎𝑛
𝑎𝑛−1
Return To Top
Key Takeaways
Proof :
Let 𝑥 be the number of circular permutations of
𝑛 distinct objects.
For each circular arrangement, the number
of linear arrangements = 𝑛
∴ The number of linear
arrangements of 𝑛 distinct objects
= 𝑛 ×
Number of circular
arrangements of 𝑛 distinct things
𝑛! = 𝑛 × 𝑥
𝑛
∴ 𝑥 =
𝑛!
= 𝑛 − 1 !
∴ The number of circular permutations of 𝑛 distinct objects is 𝑛 − 1 !
Return To Top
Key Takeaways
Circular Permutations
The number of circular permutations of 𝑛 distinct objects
is 𝑛 − 1 !
𝑎1
𝑎2
𝑎3
𝑎𝑛
𝑎𝑛−1
Difference between linear and circular arrangement :
Linear → Recognized starting place
Circular → No starting and ending place
Return To Top
Note:
If positional places are marked, then circular
arrangements can be treated as linear
arrangements.
i) There is no restriction.
9 children can be arranged in 9 − 1 ! = 8! ways
The number of ways in which 5 boys and 4 girls can be seated at a
round table, if
Return To Top
i)there is no restriction.
ii) all the girls are together
.
iii) all the girls are not together
.
iv)no two girls are not together
.
ii)All the girls are together.
Consider 4 girls as one unit.
∴ Total 6 units
6 units can be arranged in 6 − 1 ! = 5! ways.
4 girls can be arranged themselves in 4! ways.
Required number of ways = 5! × 4!
iii)All the girls are not together.
Required number of ways
= Total number of ways
− Number of ways in which all girls are
together
= 8! − 5! × 4!
Return To Top
iv)No two girls are not together.
We can arrange 5 boys in 4! ways.
There will be 5 gaps to arrange 4 girls= 4! × 5C4
Total number of ways = 4! × 5C4 × 4!
Return To Top
Out of 10 people, 5 are to be selected around a round table and the
remaining to be seated in a row. The number of arrangements is
.
Total :10 people
5! ways
Number of linear arrangements = 5!
Number of circular permutations of 𝑛 distinct
things, taken 𝑟 at a time is 𝑛𝐶𝑟 . 𝑟 − 1 !
Return To Top
Required number of circular permutations = 10𝐶5 . 5 − 1 ! = 10𝐶5 . 4!
Required number of linear arrangements = 5!
Total number of arrangements = 10𝐶5 . 4! ⋅ 5!
1
Return To Top
5
= ⋅ 10!
= 2 × 9!
Out of 10 people, 5 are to be selected around a round table and the
remaining to be seated in a row. The number of arrangements is
.
Session 06
Division and
Distribution of objects
Return To Top
Number of ways =
2
8 − 1 ! 1
2
= × 7!
= 2520
The number of ways in which a garland can be made by using 8
Return To Top
different flowers is .
Key Takeaways
2
directions are taken as not different is
1
𝑛 − 1 ! .
Circular Permutations :
Note :
Number of circular permutations of 𝑛 distinct things, if clockwise and anticlockwise
Return To Top
Example :Circular arrangement of flowers in a Garland, Beads in a Necklace.
2 𝑟
when clockwise and anticlockwise directions are same is
1
.𝑛𝐶 . 𝑟 − 1 !
2 𝑟
Required number of ways =
1
.𝑛𝐶 . 𝑟 − 1 !
1
2
18
= × 𝐶12 × 12 − 1 !
1
Return To Top
18
= × 𝐶12 × 11!
2
How many necklaces of 12 beads each can be made from 18
beads of various colors? .
Number of circular permutations of 𝑛 distinct things, taken 𝑟 at a time
Note :
Number of circular permutations of 𝑛 distinct things, taken 𝑟 at a time is 𝑛𝐶𝑟 . 𝑟 − 1 !.
In the above case, if clockwise and anticlockwise directions are taken as not different,
2 𝑟
then the number of circular permutations is
1
.𝑛𝐶 . 𝑟 − 1 !.
Return To Top
How many different necklaces be made from 10 different beads such
that three particular beads always come together.
Necklaces be made from 10 different beads =
2
8−1 !
× 3!
=
7! × 3!
2
8 different beads including 3 beads together as 1 bead 9!
2
A
7!
2
B
9!×3!
2
C
7!×3!
Return To Top
2
D
Key Takeaways
Formation of Groups:
Let us consider 4 people
Return To Top
Number of ways = 4𝐶1 × 3𝐶3 =
4!
1!3!
= 4
Key Takeaways
Formation of Groups:
𝐼) Let us divide them into two unequal groups of sizes 1 and 3
Return To Top
and 𝑛 is
𝑚+𝑛 !
𝑚! 𝑛 !
• Number of ways of dividing 𝑚 + 𝑛 + 𝑝 (𝑚 ≠ 𝑛 ≠ 𝑝) things into three unequal groups
of size 𝑚, 𝑛, 𝑝 is
𝑚+𝑛+𝑝 !
𝑚! 𝑛! 𝑝!
Key Takeaways
Results:
• Number of ways of dividing 𝑚 + 𝑛 (𝑚 ≠ 𝑛) things into two unequal groups of size 𝑚
𝑘
𝑛 is
• Number of ways of dividing 𝑛1 + 𝑛2 + ⋯ + 𝑛𝑘 into 𝑘 unequal groups of size 𝑛1, 𝑛2, … ,
(𝑛1+ 𝑛2 +⋯+ 𝑛𝑘)!
𝑛1! 𝑛2! … 𝑛𝑘!
General Results:
Return To Top
Key Takeaways
𝐼𝐼) Let us divide them into two equal groups
2!
4𝐶2×2𝐶2
Number of ways = =
4!
2!(2!∙2!)
= 3
Return To Top
• Number of ways of dividing 𝑘𝑛 things into 𝑘 equal groups of size 𝑛 each is
𝑘𝑛 !
𝑘! 𝑛! 𝑘
Key Takeaways
Results:
• Number of ways of dividing 2𝑛 things into two equal groups of size 𝑛 each is
2𝑛 !
2!(𝑛!∙𝑛!)
• Number of ways of dividing 3𝑛 things into three equal groups of size 𝑛 each is
3𝑛 !
3! 𝑛!∙𝑛!∙𝑛!
General Results:
Return To Top
𝑖)
10
2
3
5
𝑖𝑖)
Number of ways =
10!
2!3!5!
Number of ways =
10!
2! 5! 2
𝑖𝑖𝑖)
𝑖𝑣)
Number of ways =
10!
2!2! 4! 2
10!
3! 3! 31!
Consider division of 10 distinct things into groups, in different cases.
10
2
4
4
10
5 5
10
Return To Top
3
3 3
Number of ways =
1
10
2
2
2 2
2
10
1 4
10
4 6
10
2 3
Number of ways =
10!
5! 2! 5
2 3
Number of ways =
10!
1!2!3!4!
Number of ways =
10!
4!6!
2 3
Number of ways =
10!
2! 2! 2. 2! 3! 2
Return To Top
𝑣)
𝑣𝑖)
𝑣𝑖𝑖)
𝑣𝑖𝑖𝑖)
𝑖)
Number of ways =
52!
4! 13! 4
Number of ways =
52!
4! 13! 4
× 4! =
52!
13! 4
Four equal sets
𝑖𝑖) Distributing equally among four people
Four equal sets can be distributed among 4 people in 4! ways
52
Return To Top
13
13 13
13
The number of ways in which a pack of 52 cards:
𝑖) can be divided into four equal sets is .
𝑖𝑖) can be distributed among four people equally is .
Number of ways =
5!
1!2! 2! 2
= 15 Number of ways =
5!
2! 1! 23!
= 10
Case 1
Total number of ways = 15 + 10 = 25
5
1 2 2
Case 2
5
1
Return To Top
1 3
Find the number of ways in which 5 books can be
𝑖) tied up in 3 bundles.
𝑖𝑖) distributed among 3 students
such that each student get at-least one book.
𝑖) 5 books → 3 bundles
1!2! 2! 2
Number of ways =
5!
= 15 ⋅ 3! = 90
(Distribution)
Total number of ways = 90 + 60 = 150
𝑖𝑖) 5 books → 3 students
Case 1
5
1 2 2
5!
2! 1! 23!
= 10 ⋅ 3! = 60
Number of ways =
(Distribution)
Case 2
5
1
Return To Top
1 3
𝑃1 𝑃2 𝑃3 𝑃4
A boat is manned by 8 men, 4 on each side. In how many ways can 8
men be arranged if two particular men can only row on left side and
another one particular man can only row on right side?
8 men → 4 on each side
𝑃5 𝑃6 𝑃7 𝑃8
Return To Top
5!
Remaining 5 people will be divided into two groups of size 2 and 3 in ways
2!3!
Here we have to arrange the crew on both the sides, which can be done in 4! ways
5!
2!3!
Total number of arrangements = × 4! × 4! = 5760
𝑃8
𝑃7
𝑃6
𝑃5
𝑃1 𝑃2 𝑃3 𝑃4
Return To Top
Session 07
Selections from distinct and
Identical objects
Return To Top
Key Takeaways
All Possible Selections:
Case 1:
• The total number of selections out of 𝑛 distinct things taken any number of
things (including nothing)is 2𝑛
• Total number of selections = 𝑛𝐶0 + 𝑛𝐶1 + ⋯ + 𝑛𝐶𝑛 = 2𝑛
• The total number of ways of selecting at least one thing from 𝑛 distinct things
= 2𝑛 − 1
• Total number of selections = 𝑛𝐶1 + 𝑛𝐶2 + ⋯ + 𝑛𝐶𝑛 = 2𝑛 − 𝑛𝐶0 = 2𝑛 − 1
Return To Top
= 10𝐶1 + 10𝐶2 + ⋯ + 10𝐶10
= 210 − 10𝐶0
Number of ways = 210 − 1 = 1023
There are 10 lamps in a room, each can be switched on independently.
The number of ways in which the room can be illuminated is .
Return To Top
Key Takeaways
All Possible Selections:
Case 2:
• The total number of selections out of 𝑛 similar things taken any number
of things (including nothing)is 𝑛 + 1.
• The total number of ways of selecting at-least one thing from 𝑛 similar things = 𝑛.
If there are three similar letters,say A,A and A,then
Total number of selections = {no A, one A, two A’s, three A’s}= 4 ways
Case 3:
• If there are 𝑝 similar things of one type, 𝑞 similar things of second type
and 𝑟 similar things of third type, then the number of ways of selecting
atleast one thing is 𝑝 + 1 𝑞 + 1 𝑟 + 1 − 1
Return To Top
Number of ways of selecting at least one fruit
= 119
4 + 1
= 5 + 1 3 + 1 −1
Return To Top
Find the number of ways of selecting at least one fruit from 5 mangoes,
4 apples and 3 bananas.
Key Takeaways
Case 4:
• If there are 𝑝, 𝑞, 𝑟 similar things of different kinds and 𝑛 distinct things, then the
number of ways of selecting atleast one thing is 𝑝 + 1 𝑞 + 1 𝑟 + 1 2𝑛 − 1
Return To Top
2
Case II:2 alike of 1 kind and 2 alike of other kind:
4!
2!2!
Number of ways = 3𝐶 × = 18
Case III:2 alike and 2 distinct
1 2 2!
Number of ways = 3𝐶 × 6𝐶 ×
4!
= 540
Case IV:4 distinct
Number of ways = 7𝐶4 × 4! = 840
∴ Number of 4 letter words = 24 + 18 + 540 + 840 = 1422
Four letters words:
Case I:3 alike and 1 distinct.
1 3!
Number of ways = 1 × 6𝐶 ×
4!
= 24
We can get 4 four letter words, if we permutates
the letters in each of the above cases.
Number of 4 letter words with or without meaning that can be
formed using letters of the word INEFFECTIVE is:
Return To Top
Key Takeaways
MODEL BASED ON DIVISORS:
• Divisors of 12 are: 1 2 3 4 6 12
• Number of divisors = 12 Sum of divisors = 28
Let us explain these two with ‘combinations’
12 = 22 ∙ 31 (prime factorization)
Return To Top
2 = 21 ∙ 30
3 = 20 ∙ 31
4 = 22 ∙ 30
6 = 21 ∙ 31
12 = 22 ∙ 31
The number of divisors
= (Number of ways of selecting two 2’s)×
(Number of ways of selecting one 3)
= 2 + 1 1 + 1 = 6
• So, if 12 = 2𝑥 ∙ 3𝑦 then number of divisors = 𝑥 + 1 𝑦 + 1
Key Takeaways
MODEL BASED ON DIVISORS:
12 = 22 ∙ 31 (prime factorization)
1 = 20 ∙ 30
Return To Top
Sum of divisors = 20 ∙ 30 + 21 ∙ 30 + 20 ∙ 31 +22 ∙ 30 +21 ∙ 31 + 22 ∙ 31
= (20 + 21 + 22)(30 + 31)
= (7)(4)
= 28
So, if 12 = 2𝑥 ∙ 3𝑦 ,then
Sum of the divisors = (20 + 21 + 22 + ⋯ 2𝑥)(30 + 31 + 32 + ⋯ 3𝑦)
G.P
.
Return To Top
G.P.
Key Takeaways
Key Takeaways
1
𝑝𝑥1+1
−1
𝑝1−1
2
𝑝𝑥2+1
−1
𝑝2−1
⋯
• → Sum of all the divisors =
MODEL BASED ON DIVISORS:
Let 𝑁 be a positive integer and 𝑁 = 𝑝𝑥1
⋅ 𝑝𝑥2
⋅ 𝑝𝑥3
⋯
1 2 3
Where 𝑝1, 𝑝2, 𝑝3 ⋯ are distinct primes and 𝑥1, 𝑥2, 𝑥3 ⋯ ∈ 𝑁
• → Number of divisors of 𝑁 = 𝑥1 + 1 𝑥2 + 1 𝑥3 + 1
0
1 1
1
1
2
• → Sum of all the divisors = 𝑝 + 𝑝 + 𝑝 + ⋯ + 𝑝1
𝑥1
2
Return To Top
0
2
1
2
2
2
𝑥2
𝑝 + 𝑝 + 𝑝 + ⋯ 𝑝 ⋯
𝑥1 + 1 𝑥2 + 1 ⋯ , if 𝑁 is not a perfect square
MODEL BASED ON DIVISORS:
Let 𝑁 be a positive integer and 𝑁 = 𝑝𝑥1
⋅ 𝑝𝑥2
⋅ 𝑝𝑥3
⋯
1 2 3
Where 𝑝1, 𝑝2, 𝑝3 ⋯ are distinct primes and 𝑥1, 𝑥2, 𝑥3 ⋯ ∈ 𝑁
Number of ways in which 𝑁 can be resolved as a product of two factors is equal to
• 1
2
• 1
2 1
𝑥 + 1 2
Return To Top
𝑥 + 1 ⋯ + 1 , if 𝑁 is a perfect square
iii. Sum of divisors =
22−1 34−1 53−1
2−1 3−1 5−1
= 3720
iv. Sum of proper divisor = 3720 − 1 + 1350 = 2369
Return To Top
Consider the number 1350. Find
i. Number of divisors
ii. Number of proper divisors
iii. Sum of divisors
iv. Sum of proper divisors
Given: 1350 = 21 ∙ 33 ∙ 52
i. Number of divisors = 1 + 1 3 + 1 2 + 1 = 24
ii. Number of proper divisor = 24 − 2 = 22 ( Except 1 and 1350 )
In how many ways the number 8100 can be written
as product of two coprime factors?
Given: 8100 = 22 ∙ 34 ⋅ 52
Possible ways to express 8100 as product of two coprime factors are,
→ 1 × 22 ∙ 34 ∙ 52
→ 22 × 34 ∙ 52
→ 34 × (22 ∙ 52)
→ 52 × 22 ∙ 34
∴ Number of ways = 4
Return To Top
• If ′𝑛′ is the number of different prime factors of 𝑁, then the number of ways in
which 𝑁 can be resolved as product of two coprime factors is equal to 2𝑛−1.
MODEL BASED ON DIVISORS:
Return To Top
Prime Factorization of 𝑛!
Exponent of Prime in 𝑛!
• Let 𝑛! = 2𝑒1 ⋅ 3𝑒2 ⋅ 5𝑒3 ⋅ 7𝑒4 ⋯
Example:
3! = 21 ⋅ 31
4! = 23 ⋅ 31
5! = 23 ⋅ 31 ⋅ 51
100! =?
Return To Top
Key Takeaways
1
𝑒 =
𝑛
𝑝1
+
𝑛
𝑝12 +
𝑛
𝑝13 + ⋯
2
𝑒 =
𝑛
𝑝2
+
𝑛
𝑝22 +
𝑛
𝑝23 + ⋯
Exponent of 𝑝2 in 𝑛! Is given as
Exponent of Prime in 𝑛!
• Let 𝑝 be a prime number and 𝑛 be any positive integer such that :
𝑛! = 𝑝1
𝑒1 ⋅ 𝑝2
𝑒2 ⋅ 𝑝3
𝑒3 ⋯
Exponent of 𝑝1 in 𝑛! Is given as
Key Takeaways
. denotes greatest integer
Return To Top
200! = 2𝑒1 ⋅ 3𝑒2 ⋅ 5𝑒3 ⋯
1
𝑒 =
200
2
+
200
22 +
200
23 +
200
24 +
200
25 +
200
26 +
200
27 + ⋯
= 100 + 50 + 25 + 12 + 6 + 3 + 1 + 0 = 197
3
𝑒 =
200
5
+
200
52 +
200
53 +
200
54 + ⋯
= 40 + 8 + 1 + 0 = 49
200! = 2197 ⋅ 3𝑒2 ⋅ 549 ⋯
= 2148 ⋅ 249 ⋅ 3𝑒2 ⋅ 549 ⋯
= 2148 ⋅ 3𝑒2 ⋅ 1049 ⋯
Number of zeros in 200! = 49.
Find the number of zeros in 200!
Return To Top
Find the number of zeros in 200!
200
5
= 40
40
5
= 8
8
5
= 1
1
5
Return To Top
= 0
Number of zeros in 200! = 40 + 8 + 1 + 0 = 49
Alternative solution:
Number of zeros in 200! directly equals to multiples of 10 in 200!
Among the prime factors 2 and 5 the highest power of 5 in 200! will be less than
the highest power of 2 in 200!
So, the highest power of 10 in 200! will be equal to highest power of 5 in 200!
Session 08
Applications of selections in
geometry
Return To Top
HAND SHAKE
PROBLEM
Key Takeaways
GEOMETRY MODELS
• If there are 𝑛 points in a plane (no three are collinear)
then by joining them we can obtain:
i. 𝑛 𝐶2 straight lines
ii.𝑛𝐶3 triangles
𝑃1 𝑃2
𝑃3 𝑃4
Return To Top
Key Takeaways
GEOMETRY MODELS
• If there are 𝑛 points in a plane of which 𝑚 points are
collinear, then by joining them, we can obtain
i. 𝑛 𝐶2 − 𝑚𝐶2 + 1 straight lines
ii.𝑛𝐶3 − 𝑚𝐶3 triangles
Return To Top
• 𝑛𝐶2 − 𝑛 = 𝑛 𝑛−3
2
Key Takeaways
GEOMETRY MODELS
The number of diagonals of a polygon of 𝑛 sides (𝑛 vertices) is
Return To Top
𝑚𝐶2 × 𝑛𝐶2
Key Takeaways
GEOMETRY MODELS
If a set of 𝑚 parallel lines are intersected by another set of 𝑛 parallel
lines, then the number of parallelograms, we can obtain is
•
Return To Top
𝑛𝐶2
i. 𝑛 straight lines in a plane is
ii. 𝑛 circles in a plane is 2 × 𝑛 𝐶2
Key Takeaways
GEOMETRY MODELS
The maximum number of points of intersection of
Return To Top
There are 12 points in a place of which 7 are collinear.
By joining them,we can have :
Return To Top
i. Number of straight lines
ii. Number of triangles
iii.Number of circles
iv.Number of pentagons
i. Number of straight lines = 12𝐶2 − 7𝐶2 + 1 = 46
ii. Number of triangles = 12𝐶3 − 7𝐶3 = 185
iii. Number of circles = 12𝐶3 − 7𝐶3 = 185
iv. Number of pentagons = 12𝐶5 − 7𝐶5 = 771
The maximum number of points of intersection of :
Return To Top
i)20 lines in a plane is .
ii) 10 circles in a plane is .
iii)20 lines and 10 circles in a plane is .
i)20 line in a plane
The maximum number of points of intersection of 𝑛 straight lines in a plane is 𝑛 𝐶2
⇒ Maximum number of points of intersection of 20 straight lines
= 20𝐶2 = 190
The maximum number of points of intersection of 𝑛 circles in a plane is 2 × 𝑛𝐶2.
⇒ Maximum number of points of intersection of 10 circles
= 2 × 10𝐶2 = 2 × 45 = 90
Case 𝑖
= 2 × 20𝐶1 × 20𝐶1 = 400
Case 𝑖𝑖
= 2 × 10𝐶2 = 90
Case 𝑖𝑖𝑖
Return To Top
= 2 × 10𝐶2 = 90
Total = 400 + 90 + 190
= 680
A convex polygon has 44 diagonals. Find the number of its sides.
⋮
⋮
⋮
44 diagonals
Let 𝑛 be the number of slides ⇒ 𝑛 points (vertices)
The number of diagonals of a polygon of 𝑛 sides (𝑛
vertices) is 𝑛𝐶2 − 𝑛.
⋮
𝑉1 𝑉2
𝑉𝑛
𝑛𝐶2 − 𝑛 =
2
𝑛 (𝑛−1)
− 𝑛 = 44
⇒ 𝑛 (𝑛−3)
= 44
2
⇒ 𝑛 𝑛 − 3 = 88
⇒ 𝑛2 − 3𝑛 − 88 = 0
𝑉3
⇒ 𝑛 − 11 𝑛 + 8 = 0
Return To Top
⇒ 𝑛 = 11 (∵ 𝑛 = −8 is not possible)
𝐵 𝐶
Triangle can be formed by
𝐴
𝐴𝐵 → 3 points 𝐵𝐶 → 4 points 𝐴𝐶 → 5 points
Selecting one point each on one side
Selecting two points on one side and
third point on any other side
OR
In a triangle 𝐴𝐵𝐶, the sides 𝐴𝐵, 𝐵𝐶 and 𝐴𝐶 have 3, 4 and 5 points
respectively on them.The number of triangles that can be
constructed using these points as vertices is .
Return To Top
𝐵 𝐶
Case (i)
Selecting one point each on = 3𝐶1 × 4𝐶1 × 5𝐶1 = 60
one side
Case (ii)
Selecting two points on one side
and third point on any other side
= 3𝐶2 × 9𝐶1 + 4𝐶2 × 8𝐶1 + 5𝐶2 × 7𝐶1
= 27 + 48 + 70 = 145
∴ Number of triangles = 60 + 145 = 205
Return To Top
𝑛 points
Given : 𝑇𝑛 +1 − 𝑇𝑛 = 21
Number of triangles = 𝑇𝑛 = 𝑛 𝐶3, 𝑇𝑛 +1 = 𝑛 +1𝐶3
𝑇𝑛+1 − 𝑇𝑛 = 21
𝑛𝐶2 + 𝑛𝐶3 = 𝑛+1𝐶3
⇒ 𝑛+1𝐶3 − 𝑛𝐶3 = 21
⇒ 𝑛𝐶2 = 21
⇒ 𝑛 𝑛 − 1 = 42
⇒ 𝑛 = 7
Let 𝑇𝑛 denotes the number of triangles which can be formed
by joining the 𝑛 points which lie on a circle. If 𝑇𝑛+1 − 𝑇𝑛= 21,
then the value of 𝑛 is .
Return To Top
Given : A regular polygon of 10 sides
two vertices are consecutive
Number of ways in which no =
Number of ways of selecting any 3 vertices.
−
Number of ways of selecting 3 consecutive vertices.
−
Number of ways of selecting 3 vertices such
that two are consecutive.
A regular polygon of 10 sides is constructed. In how many ways can 3
vertices be selected so that no two vertices are consecutive ?
Return To Top
Let the vertices be :𝑉1, 𝑉2, 𝑉3, ⋯ , 𝑉10
Number of ways of selecting any 3 vertices = 10𝐶3
𝑉1 𝑉2 𝑉3, 𝑉2 𝑉3 𝑉4 , ⋯ , 𝑉10 𝑉1 𝑉2
𝑉2
𝑉1
𝑉3
𝑖) Starting with 𝑉 𝑉 = 6
𝑉1 𝑉2 𝑉4
𝑉7
𝑉8
𝑉9
6 ways
𝑉6
𝑉5
Return To Top
𝑉4
𝑉5
𝑉6
𝑉7
𝑉8
𝑉9
𝑉10
1 2
Similarly,we have starting with
𝑉2 𝑉3, 𝑉3 𝑉4, 𝑉4𝑉5, ⋯ , 𝑉10 𝑉1
Number of ways of selecting 3 consecutive vertices = 10
Number of ways of selecting 3 consecutive vertices
such that two are consecutive = 10 × 6 = 60
Number of ways of selecting any 3 vertices = 10𝐶3
Number of ways of selecting 3 consecutive vertices = 10
Number of ways of selecting 3 vertices
such that two are consecutive = 60
Number of ways in which no two vertices are consecutive
= 10𝐶3 − 10 − 60
= 120 − 70
= 50
𝑉
Return To Top
2
𝑉1
𝑉3
𝑉4
𝑉5
𝑉6
𝑉7
𝑉8
𝑉9
𝑉10
How many squares are there in a chess board?
A chessboard has 64 small squares.
There are many more different sized
squares on the chessboard.
Return To Top
1. No.of 8 × 8 𝑠𝑞. = 1
2. No.of 7 × 7 𝑠𝑞. = 4
3. No.of 6 × 6 𝑠𝑞. = 9
4. No.of 5 × 5 𝑠𝑞. = 16
5. No.of 4 × 4 𝑠𝑞. = 25
6. No.of 3 × 3 𝑠𝑞. = 36
7. No.of 2 × 2 𝑠𝑞. = 49
8. No.of 1 × 1 𝑠𝑞. = 64
∴ Total number of squares on chessboard
⇒ 64 + 49 + 36 + 25 + 16 + 9 + 4 + 1 = 204
Given word : ROORKEE
Let A ≡ {words begin with R }
B ≡ {words end with E }
A’ ∩ B’ ≡ {words neither begin with R nor end with E }
ROORKEE 𝑛 𝑈 =
7!
2! . 2! . 2!
𝑛 𝐴 =
6!
2! . 2!
R
Find the number of words that can be made by permutating all
the letters of the word ROORKEE, which neither begins with R nor
end with E.
Return To Top
Let A ≡ {words begin with R } B ≡ {words end with E }
A’ ∩ B’ ≡ {words neither begin with R nor end with E }
ROORKEE 𝑛 𝑈 =
7!
2! . 2! . 2!
𝑛 𝐴 =
6!
2! . 2!
R
E
𝑛 𝐵 =
6!
2! . 2!
𝑛 𝐴 ∩ 𝐵
5!
2!
R E
=
Return To Top
ROORKEE
𝑛 𝐴 =
6!
2! . 2!
𝑛 𝐵 =
6!
2! . 2!
𝑛 𝐴 ∩ 𝐵 =
5!
2!
𝑛 𝐴′ ∩ 𝐵′
= 𝑛 𝑈
= 𝑛 𝑈 − 𝑛 𝐴 𝖴 𝐵
− 𝑛 𝐴 − 𝑛 𝐵 + 𝑛 𝐴 ∩ 𝐵
=
7! 6!
− −
6!
+ 5!
2! .2! .2! 2! . 2! 2! . 2! 2!
𝑛 𝑈 =
7!
Return To Top
2! . 2! . 2!
= 630 − 180 − 180 + 60 = 330
Session 09
Distribution of objects to
distinct boxes
Return To Top
Key Takeaways
𝑛
𝐷 = 𝑛! 1 −
1
+
1
−
1
+ ⋯ + −1 𝑛 1
1! 2! 3! 𝑛 !
where 𝑛 ≥ 2
Derangements
If 𝑛 things are arranged in a row, then the number of rearrangements such that none of
them occupy their original positions are called Dearangements.
The number of Derangements of 𝑛 distinct things can be denoted by 𝐷𝑛 .
Return To Top
Given word : BAG
All permutations = 3! = 6 ways =
ABG
AGB
BAG
BGA
GAB
GBA
Number of derangements = 𝐷3 = 2
Find the number of derangements of the letters of the word BAG.
𝑛
𝐷 = 𝑛! 1 −
1
+
1
−
1
+ ⋯ + −1 𝑛 1
1! 2! 3! 𝑛 !
where 𝑛 ≥ 2
3 1! 2! 3!
𝐷 = 3! 1 −
1
+
1
−
1
= 2
Using formula:
Return To Top
Six letters with numbers 1,2,3,4,5,6written on them are put in six
envelopes with numbers 1,2,3,4,5,6written on them too.
𝑖) No letter is in its correct envelope is
𝑖𝑖) At least one letter is in its correct envelope is
𝑖𝑖𝑖) At most two letters are not in their correct envelopes is
𝑖𝑣) Exactly three letters are not in their correct envelopes is
Given:
.
.
Lette
r 1
.
Lette
r 2 Lette
r 3 Lette
r 4 Lette
r 5
. . . . . .
Return To Top
. . . . .
. . . .
Lette
r 6
1 2 3 4 5 6
𝑖) No letter is in its correct envelope is
.
.
Lette
r 1
.
Lette
r 2 Lette
r 3 Lette
r 4 Lette
r 5
. . . . . .
Return To Top
. . . . .
. . . .
Lette
r 6
1 2 3 4 5 6
Number of ways = 𝐷6= 265
𝑖𝑖) At least one letter is in its correct envelope is
Number of ways =Total number of permutations
− No letter is in its correct envelope
= 6! − 265
= 455
𝑖𝑖𝑖) At most two letters are not in their correct envelopes is
+ +
All letters are
in its correct
envelope
One letter isn’t
in its correct
envelope
Two letters are
not in their
correct envelope
= 1 + 0 + 6𝐶2 × 𝐷2
= 16
𝑖𝑣) Exactly three letters are not in their correct envelopes is
= 6𝐶3 × 𝐷3
= 40
Return To Top
Principle of Inclusion and Exclusion
𝑛 𝐴 𝖴 𝐵
= + -
𝑛 𝐴 𝑛 𝐵 𝑛 𝐴 ∩ 𝐵
include exclude
𝑛 𝐴 𝖴 𝐵 = 𝑛 𝐴 + 𝑛 𝐵 − 𝑛 𝐴 ∩ 𝐵
Return To Top
𝐴 𝐵
𝐶
𝑛 (𝐵 ∩ 𝐶)
𝑛 (𝐴 ∩ 𝐵 ∩ 𝐶)
𝑛 𝐴 𝖴 𝐵 𝖴 𝐶
= 𝑛 (𝐴) + 𝑛 (𝐵) + 𝑛 (𝐶) − 𝑛 (𝐴 ∩ 𝐵) − 𝑛 (𝐴 ∩ 𝐶) −
+
include
include
exclude
Principle of Inclusion and Exclusion
Return To Top
In general, 𝑛 𝐴1 𝖴 𝐴2 𝖴 𝐴3 𝖴 ⋯ 𝖴 𝐴𝑛
= ෍ 𝑛 𝐴𝑖 − ෍ 𝑛 𝐴𝑖 ∩ 𝐴𝑗 + ෍ 𝑛 𝐴𝑖 ∩ 𝐴𝑗 ∩ 𝐴𝑘 − ⋯ + −1 𝑛 𝑛 𝐴1 ∩ 𝐴2 ∩ 𝐴3 ∩ ⋯ ∩ 𝐴𝑛
⇒ 𝑛 𝐴1′ 𝖴 𝐴2′ 𝖴 𝐴3′ 𝖴 ⋯ 𝖴 𝐴𝑛′ = 𝑛 𝑈 − 𝑛 𝐴1 𝖴 𝐴2 𝖴 𝐴3 𝖴 ⋯ 𝖴 𝐴𝑛
Return To Top
Principle of Inclusion and Exclusion
Key Takeaways
Distinct Thing → Distinct Boxes:
The number of ways of distributing 𝑛 distinct things in 𝑟 distinct boxes such
that each box is filled with
Case 1
0 or more things (i.e. empty boxes are allowed) = 𝑟𝑛
Case 2
Atleast one thing (i.e. empty boxes are not allowed)
= 𝑟𝑛 − 𝑟 𝐶1 𝑟 − 1 𝑛 + 𝑟 𝐶2 𝑟 − 2 𝑛 − ⋯
+
−1 𝑟−1 ∙ 𝑟𝐶𝑟−1 1 𝑛
Return To Top
In how many ways 5 different balls can be distributed into 3 boxes so
that no box remains empty.
= 35 − 3𝐶1 3 − 1 5 + 3𝐶2 3 − 2 5 − 3𝐶3 3 − 3 5
= 150
The required number of ways
Atleast one ball (i.e. empty boxes are not allowed)
−1 𝑟−1 ∙ 𝑟𝐶𝑟−1 1 𝑛
Return To Top
= 𝑟𝑛 − 𝑟𝐶1 𝑟 − 1 𝑛 + 𝑟𝐶2 𝑟 − 2 𝑛 − ⋯ +
where 𝑛 = 5, 𝑟 = 3
𝑖𝑖) Number of functions from 𝐴 to 𝐵 such that
every element in 𝐵 has atleast one pre-image
If 𝐴 = 𝑎1, 𝑎2, 𝑎3, 𝑎4, 𝑎5 and 𝐵 = 𝑏1, 𝑏2, 𝑏3 then find
𝑖) Number of functions from 𝐴 to 𝐵
= 𝑟𝑛 = 35 = 243
𝑩
𝑨
𝒃𝟏
𝒃𝟐
𝒃𝟑
= 𝑟𝑛 − 𝑟𝐶1 𝑟 − 1 𝑛 + 𝑟𝐶2 𝑟 − 2 𝑛 − ⋯ +
= 35 − 3𝐶1 3 − 1 5 + 3𝐶2 3 − 2 5
= 150
−1 𝑟−1 ∙ 𝑟𝐶𝑟−1 1 𝑛
𝒂𝟏
𝒂𝟐
𝒂𝟑
𝒂𝟒
𝒂𝟓
If 𝐴 = 𝑎1,𝑎2, 𝑎3, 𝑎4, 𝑎5 and 𝐵 = 𝑏1,𝑏2, 𝑏3 then find
Return To Top
𝑖) Number of functions from 𝐴 to 𝐵
𝑖𝑖) Number of functions from 𝐴 to 𝐵 such that every element in
𝐵 has atleast one pre-image
Key Takeaways
Proof:
Let 𝑛 identical objects put on floor
1 2 3 4 𝑛 − 1
⋯ ⋯ 𝑛
Identical Things → Distinct Boxes:
The number of ways of distributing 𝑛 distinct things in 𝑟 distinct boxes such
that each box is filled with
Return To Top
Case 1
0 or more things (i.e. empty boxes are allowed) = 𝑛+𝑟−1𝐶𝑟−1
𝑟 − 1 identical partitions
𝑛 identical objects 𝑟 − 1 identical partitions
𝑛! 𝑟−1 !
𝑛+𝑟−1 !
= 𝑛+𝑟−1𝐶𝑟 −1
Proof (Cont.⋯):
To form 𝑟 −groups we need
1 2 3 4 ⋯ ⋯ 𝑛 − 1
Return To Top
𝑛
Key Takeaways
𝑛−1𝐶𝑟−1
0 or more things (i.e.empty boxes are allowed) = 𝑛+𝑟−1𝐶𝑟−1
Now where 𝑛 − 𝑟 objects to be distributed in 𝑟 boxes.
= 𝑛−𝑟+𝑟−1𝐶𝑟−1 = 𝑛−1𝐶𝑟−1
Key Takeaways
Return To Top
Identical Things → Distinct Boxes:
The number of ways of distributing 𝑛 distinct things in 𝑟 distinct boxes such
that each box is filled with
Case 2
Atleast one thing (i.e. empty boxes are not allowed) =
Proof:
Let us give each box 1object to satisfy atleast one condition.
Key Takeaways
Identical Things → Distinct Boxes:
For the equation 𝑥1 + 𝑥2 + 𝑥3 + ⋯ + 𝑥𝑟 = 𝑛
Where 𝑛 is 𝑛 identical units of 1.
Return To Top
𝑖) Number of non-negative integral solution 𝑛 +𝑟 −1𝐶𝑟 −1
𝑖𝑖) Number of positive integral solution 𝑛−1𝐶𝑟−1
In how many ways 5 identical balls can be distributed into 3 distinct
boxes so that no box remains empty.
Return To Top
Given:
𝑛 = 5, 𝑟 = 3
The required number of ways
= 𝑛−1𝐶𝑟−1
= 4𝐶2
= 6
For the equation 𝑎 + 𝑏 + 𝑐 + 𝑑 = 12.
Return To Top
𝑖) Number of non-negative integral solutions are
𝑖𝑖) Number of positive integral solutions are
𝑖) Number of non-negative integral solutions are
Given:
𝑛 = 12, 𝑟 = 4
𝑛+𝑟−1𝐶𝑟−1 = 12+4−1𝐶4−1
= 15𝐶3 = 455
𝑖𝑖) Number of positive integral solutions are
𝑛−1𝐶𝑟−1
= 12−1𝐶4−1
= 11𝐶3 = 165
In how many ways 10 apples, 5 mangoes, 4 oranges can be distributed
among 4 persons, when each person may receive any number of
fruits. (Fruits of each type are identical)
Return To Top
Given:
Number of persons = 4
Let 𝑥1, 𝑥2, 𝑥3, 𝑥4 be number of apples receive by 𝑃1,𝑃2,𝑃3,𝑃4respectively
∴ 𝑥1 + 𝑥2 + 𝑥3 + 𝑥4 = 10
10+4−1𝐶4−1
= 13𝐶3
Let 𝑦1, 𝑦2, 𝑦3, 𝑦4 be number of mangoes receive by 𝑃1,𝑃2,𝑃3,𝑃4respectively
∴ 𝑦1 + 𝑦2 + 𝑦3 + 𝑦4 = 5
5+4−1𝐶4−1
= 8𝐶3
Let 𝑧1, 𝑧2, 𝑧3, 𝑧4 be number of oranges receive by 𝑃1,𝑃2,𝑃3,𝑃4 respectively
∴ 𝑧1 + 𝑧2 + 𝑧3 + 𝑧4 = 4
4+4−1𝐶4−1
= 7𝐶3
The required number of ways
= 13𝐶3 × 8𝐶3 × 7𝐶3
= 560560
Return To Top
In how many ways 10 apples, 5 mangoes, 4 oranges can be distributed
among 4 persons, when each person may receive any number of
fruits. (Fruits of each type are identical)
Multinomial theorem and
its applications
Return To Top
Session 10
16
3 3 3 3
4
𝑎 𝑏 𝑐 𝑑
𝑎 + 𝑏 + 𝑐 + 𝑑 = 4
4+4−1𝐶4−1
= 7𝐶3
= 35
Given:
Remaining 4 to be distributed among
4 Persons so that any one receive any
amount,which is equal to the number of
non-negative integral solutions of
In how many ways can Rs. 16 be divided among 4 persons such that
none of them gets less than Rs. 3.
Return To Top
Find the number of non-negative integral solutions of
𝑎 + 𝑏 + 𝑐 + 𝑑 ≤ 20.
Return To Top
Given: 𝑎 + 𝑏 + 𝑐 + 𝑑 ≤ 20 ⋯ (𝑖)
Let 𝑒 ≥ 0 be a dummy variable such that
𝑎 + 𝑏 + 𝑐 + 𝑑 + 𝑒 = 20 ⋯ (𝑖𝑖)
Now, number of non-negative integral solutions of (𝑖)
= number of non-negative integral solutions of (𝑖𝑖)
= 𝑛+𝑟−1𝐶𝑟−1
𝑛 = 20, 𝑟 = 5
= 20+5−1𝐶5−1
𝑥 + 𝑦 + 𝑧 = 20
𝑢 − 4 + 𝑣 + 1 + 𝑧 = 20
𝑢 + 𝑣 + 𝑧 = 23⋯ (𝑖𝑖)
where 𝑢 ≥ 0, 𝑣 ≥ 0 and 𝑧 ≥ 0
Given: 𝑥 + 𝑦 + 𝑧 = 20 ⋯ (𝑖)
𝑥 ≥ −4 ⇒ 𝑥 + 4 ≥ 0
𝑦 ≥ 1 ⇒ 𝑦 − 1 ≥ 0
Let,𝑥 + 4 = 𝑢 and 𝑦 − 1 = 𝑣
Number of required solutions of (𝑖)
= Number of non- negative integral solutions of (𝑖𝑖)
= 23+3−1𝐶3−1 = 25𝐶2 = 300
Find the number of integral solutions of
𝑥 + 𝑦 + 𝑧 = 20 where 𝑥 ≥ −4, 𝑦 ≥ 1, ≥ 𝑧 ≥ 0.
Return To Top
Key Takeaways
Application of Multinomial Theorem
In this theorem we try to write all possible outcomes in powers of a random
variable 𝑥 and then calculate coefficient of required outcomes.
Return To Top
Find the number of ways of distributing 10 identical apples
among 3 children without any restrictions
Let 𝑎, 𝑏, 𝑐 be number of apples distributed to three children
𝑎 + 𝑏 + 𝑐 = 10; 𝑎 ≥ 0, 𝑏 ≥ 0. 𝑐 ≥ 0
Possible outcomes = 𝑎
𝑥0 + 𝑥1 + 𝑥2 + ⋯
𝑏
𝑥0 + 𝑥1 + 𝑥2 + ⋯
𝑐
𝑥0 + 𝑥1 + 𝑥2 + ⋯
Number of solution will be = Coefficient of 𝑥10 in
𝑥0 + 𝑥1 + 𝑥2 + ⋯ 𝑥0 + 𝑥1 + 𝑥2 + ⋯ 𝑥0 + 𝑥1 + 𝑥2 + ⋯
Infinite G.P
.
Return To Top
Coefficient of 𝑥𝑟 in 1 − 𝑥 −𝑚 is 𝑚+𝑟−1𝐶𝑟
Coefficient of 𝑥10 in
1 1 1
1−𝑥 1−𝑥 1−𝑥
= Coefficient of 𝑥10 in 1 − 𝑥 −3
𝑎
𝑆∞ = 1−𝑟
= 66
= 10+3−1𝐶10
= 12𝐶10
𝑚 = 3, 𝑟 = 10
Return To Top
Find the number of non-negative integral solutions of
𝑥1 + 𝑥2 + 𝑥3 + 4𝑥4 = 20.
Given: 𝑥1+𝑥2 + 𝑥3 + 4𝑥4 = 20
Number of non-negative integral solutions
= the coefficient of 𝑥20 in the product 1 + 𝑥 + 𝑥2 + ⋯ 3
1 + 𝑥4 + 𝑥8 + ⋯
𝑚+1 𝐶2 𝑥2 − 𝑚+2 𝐶3 𝑥3 + ⋯
= coefficient of 𝑥20 in 1 − 𝑥 −3 1 − 𝑥4 −1 1 + 𝑥 −𝑚
= 1 − 𝑚𝐶1 𝑥 +
= Coefficient of 𝑥20 in
(1 + 3𝐶1𝑥 + 4𝐶2𝑥2 + ⋯ + 𝑟+2𝐶𝑟𝑥𝑟) 1 + 𝑥4 + 𝑥8 + ⋯
= 1 + 6𝐶4 + 10𝐶8 + 14𝐶12 + 18𝐶16 + 22𝐶20
= 536
Return To Top
Given :
15days → to visit for 4 days,
No two visits are consecutive.
Let 𝑉1, 𝑉2, 𝑉3, 𝑉4 be the 4 visiting days
An engineer is required to visit a factory for exactly 4 days during the
first 15 days of every month and it is mandatory that no two visits
take place on consecutive days. Then the number of all possible ways
in which such visits to the factory can be made by the engineer
during 1-15June 2021 is .
Return To Top
JEE (ADVANCED)2020
= 8+5−1𝐶5−1 = 12𝐶4
= 495
𝑎 𝑏 𝑐 𝑑
𝑉1 𝑉2 𝑉3 𝑉4
Return To Top
𝑒
where, 𝑎, 𝑏, 𝑐, 𝑑, 𝑒 be the remaining days
So, 𝑎 + 𝑏 + 𝑐 + 𝑑 + 𝑒 = 11 and 𝑎 ≥ 0, 𝑏 ≥ 1, 𝑐 ≥ 1, 𝑑 ≥ 1, 𝑒 ≥ 0
Put 𝑏 = 1 + 𝑥, 𝑐 = 1 + 𝑦, 𝑑 = 1 + 𝑡.
then, 𝑎 + 𝑥 + 𝑦 + 𝑡 + 𝑒 = 8 and 𝑎 ≥ 0, 𝑥 ≥ 0, 𝑦 ≥ 0, 𝑡 ≥ 0, 𝑒 ≥ 0
Number of non-negative integral solution
= 𝑛+𝑟−1𝐶𝑟−1
Let 𝑉1, 𝑉2, 𝑉3, 𝑉4 be the 4 visiting days
Consider,
Key Takeaways
Result :
If 𝑑1, 𝑑2, ⋯ , 𝑑𝑛 are the given non-zero digits, then the sum of all 𝑛 digit numbers
(without repetition) is equal to
𝑛 − 1 ! (𝑑1 + 𝑑2 + ⋯ + 𝑑𝑛 ) (100 + 101 + ⋯ + 10𝑛 −1)
Return To Top
Find the sum of all four-digit numbers formed by using the
digits 2, 3, 4,5 without repetition.
_
Given digits:2, 3, 4, 5
_ _ _ _
Numbers having 2 in unit’s place = 3! 5
4
3
2
3!
Similarly,Numbers having 3, 4, 5 in
units place = 3!
_ _ _ _ 5
4
3
2
3!
Sum of all digits in unit’s place = 3! 2 + 3 + 4 + 5
Return To Top
Number of 4-digit numbers = 4! = 24 _ _ _
Click to add tex4
t!
_ _ _ _
2
Sum of all digits in unit’s place = 3!
Number having 2 in ten’s place= 3!
2 + 3 + 4 + 5
3
4
Similarly,Numbers having 3, 4, 5 in
5
ten’s place = 3! _ _ _ _
Sum of digits in ten’s place =3! 2 + 3 + 4 + 5 × 10
Sum of all digits = Sum of all digits of unit’s ,ten’s, hundred’s and
thousand’s places.
Hence, the sum of all 4 digited number
= 3! 2 + 3 + 4 + 5 1111
Return To Top
= 3! 2 + 3 + 4 + 5 100 + 101 + 102 + 103
= 6 × 14 × 1111 = 93324
Return To Top
THANK
YOU
Return To Top

More Related Content

Similar to unit 4 Elementary descrete mathematics combinatorics.pptx

Lesson 1 student notes
Lesson 1 student notesLesson 1 student notes
Lesson 1 student notes
Joanne Rosa Crooks
 
Section 4.1 And 4.2 Plus Warm Ups
Section 4.1 And 4.2 Plus Warm UpsSection 4.1 And 4.2 Plus Warm Ups
Section 4.1 And 4.2 Plus Warm Ups
Jessca Lundin
 
Real numbers system
Real numbers systemReal numbers system
Real numbers system
Pradeep Agrawal
 
order of operations
order of operationsorder of operations
order of operations
Mary Ann Joy Cedillo
 
NLC Math 8_I_Lesson 33.pptx
NLC Math 8_I_Lesson 33.pptxNLC Math 8_I_Lesson 33.pptx
NLC Math 8_I_Lesson 33.pptx
HezelGraceAquino1
 
Permutations and Combinations
Permutations and CombinationsPermutations and Combinations
Permutations and Combinations
Angel Willis
 
G10M-Q3-L1-Permutation-of-Objects-Grade 10.pptx
G10M-Q3-L1-Permutation-of-Objects-Grade 10.pptxG10M-Q3-L1-Permutation-of-Objects-Grade 10.pptx
G10M-Q3-L1-Permutation-of-Objects-Grade 10.pptx
KirbyRaeDiaz2
 
P1-Chp13-Integration.pptx
P1-Chp13-Integration.pptxP1-Chp13-Integration.pptx
P1-Chp13-Integration.pptx
YashTiwari512330
 
W1-L1 Negative-numbers-ppt..pptx
W1-L1 Negative-numbers-ppt..pptxW1-L1 Negative-numbers-ppt..pptx
W1-L1 Negative-numbers-ppt..pptx
Ghassan44
 
Grade 10 Permutation PowerPoint Presentation
Grade 10 Permutation PowerPoint PresentationGrade 10 Permutation PowerPoint Presentation
Grade 10 Permutation PowerPoint Presentation
Melvin Verdadero
 
powerpoints probability.pptx
powerpoints probability.pptxpowerpoints probability.pptx
powerpoints probability.pptx
carrie mixto
 
Semana 10 numeros complejos i álgebra-uni ccesa007
Semana 10   numeros complejos i álgebra-uni ccesa007Semana 10   numeros complejos i álgebra-uni ccesa007
Semana 10 numeros complejos i álgebra-uni ccesa007
Demetrio Ccesa Rayme
 
Lesson 33 Powerpoint
Lesson 33 PowerpointLesson 33 Powerpoint
Lesson 33 Powerpoint
Randall Micallef
 
Yr7-Sequences.pptx
Yr7-Sequences.pptxYr7-Sequences.pptx
Yr7-Sequences.pptx
FrancaOkechukwu
 
Whiz Kid 4th Grade Math
Whiz Kid 4th Grade MathWhiz Kid 4th Grade Math
Whiz Kid 4th Grade Math
Terrell Johnson
 
P2-Chp3-SequencesAndSeries from pure maths 2.pptx
P2-Chp3-SequencesAndSeries from pure maths 2.pptxP2-Chp3-SequencesAndSeries from pure maths 2.pptx
P2-Chp3-SequencesAndSeries from pure maths 2.pptx
ArafathAliMathsTeach
 
1.6 multiplication i w
1.6 multiplication i w1.6 multiplication i w
1.6 multiplication i w
Tzenma
 
Introduction to fractions and concepts
Introduction to fractions and conceptsIntroduction to fractions and concepts
Introduction to fractions and concepts
Martha Ardila Ibarra
 
6. Permutations and Combinations-Revised (1).pptx
6. Permutations and Combinations-Revised (1).pptx6. Permutations and Combinations-Revised (1).pptx
6. Permutations and Combinations-Revised (1).pptx
TonmoyKabiraj
 
Module 3 topic 2 notes
Module 3 topic 2 notesModule 3 topic 2 notes
Module 3 topic 2 notes
cauleyc
 

Similar to unit 4 Elementary descrete mathematics combinatorics.pptx (20)

Lesson 1 student notes
Lesson 1 student notesLesson 1 student notes
Lesson 1 student notes
 
Section 4.1 And 4.2 Plus Warm Ups
Section 4.1 And 4.2 Plus Warm UpsSection 4.1 And 4.2 Plus Warm Ups
Section 4.1 And 4.2 Plus Warm Ups
 
Real numbers system
Real numbers systemReal numbers system
Real numbers system
 
order of operations
order of operationsorder of operations
order of operations
 
NLC Math 8_I_Lesson 33.pptx
NLC Math 8_I_Lesson 33.pptxNLC Math 8_I_Lesson 33.pptx
NLC Math 8_I_Lesson 33.pptx
 
Permutations and Combinations
Permutations and CombinationsPermutations and Combinations
Permutations and Combinations
 
G10M-Q3-L1-Permutation-of-Objects-Grade 10.pptx
G10M-Q3-L1-Permutation-of-Objects-Grade 10.pptxG10M-Q3-L1-Permutation-of-Objects-Grade 10.pptx
G10M-Q3-L1-Permutation-of-Objects-Grade 10.pptx
 
P1-Chp13-Integration.pptx
P1-Chp13-Integration.pptxP1-Chp13-Integration.pptx
P1-Chp13-Integration.pptx
 
W1-L1 Negative-numbers-ppt..pptx
W1-L1 Negative-numbers-ppt..pptxW1-L1 Negative-numbers-ppt..pptx
W1-L1 Negative-numbers-ppt..pptx
 
Grade 10 Permutation PowerPoint Presentation
Grade 10 Permutation PowerPoint PresentationGrade 10 Permutation PowerPoint Presentation
Grade 10 Permutation PowerPoint Presentation
 
powerpoints probability.pptx
powerpoints probability.pptxpowerpoints probability.pptx
powerpoints probability.pptx
 
Semana 10 numeros complejos i álgebra-uni ccesa007
Semana 10   numeros complejos i álgebra-uni ccesa007Semana 10   numeros complejos i álgebra-uni ccesa007
Semana 10 numeros complejos i álgebra-uni ccesa007
 
Lesson 33 Powerpoint
Lesson 33 PowerpointLesson 33 Powerpoint
Lesson 33 Powerpoint
 
Yr7-Sequences.pptx
Yr7-Sequences.pptxYr7-Sequences.pptx
Yr7-Sequences.pptx
 
Whiz Kid 4th Grade Math
Whiz Kid 4th Grade MathWhiz Kid 4th Grade Math
Whiz Kid 4th Grade Math
 
P2-Chp3-SequencesAndSeries from pure maths 2.pptx
P2-Chp3-SequencesAndSeries from pure maths 2.pptxP2-Chp3-SequencesAndSeries from pure maths 2.pptx
P2-Chp3-SequencesAndSeries from pure maths 2.pptx
 
1.6 multiplication i w
1.6 multiplication i w1.6 multiplication i w
1.6 multiplication i w
 
Introduction to fractions and concepts
Introduction to fractions and conceptsIntroduction to fractions and concepts
Introduction to fractions and concepts
 
6. Permutations and Combinations-Revised (1).pptx
6. Permutations and Combinations-Revised (1).pptx6. Permutations and Combinations-Revised (1).pptx
6. Permutations and Combinations-Revised (1).pptx
 
Module 3 topic 2 notes
Module 3 topic 2 notesModule 3 topic 2 notes
Module 3 topic 2 notes
 

Recently uploaded

বাংলাদেশ অর্থনৈতিক সমীক্ষা (Economic Review) ২০২৪ UJS App.pdf
বাংলাদেশ অর্থনৈতিক সমীক্ষা (Economic Review) ২০২৪ UJS App.pdfবাংলাদেশ অর্থনৈতিক সমীক্ষা (Economic Review) ২০২৪ UJS App.pdf
বাংলাদেশ অর্থনৈতিক সমীক্ষা (Economic Review) ২০২৪ UJS App.pdf
eBook.com.bd (প্রয়োজনীয় বাংলা বই)
 
A Independência da América Espanhola LAPBOOK.pdf
A Independência da América Espanhola LAPBOOK.pdfA Independência da América Espanhola LAPBOOK.pdf
A Independência da América Espanhola LAPBOOK.pdf
Jean Carlos Nunes Paixão
 
Assessment and Planning in Educational technology.pptx
Assessment and Planning in Educational technology.pptxAssessment and Planning in Educational technology.pptx
Assessment and Planning in Educational technology.pptx
Kavitha Krishnan
 
CACJapan - GROUP Presentation 1- Wk 4.pdf
CACJapan - GROUP Presentation 1- Wk 4.pdfCACJapan - GROUP Presentation 1- Wk 4.pdf
CACJapan - GROUP Presentation 1- Wk 4.pdf
camakaiclarkmusic
 
Pride Month Slides 2024 David Douglas School District
Pride Month Slides 2024 David Douglas School DistrictPride Month Slides 2024 David Douglas School District
Pride Month Slides 2024 David Douglas School District
David Douglas School District
 
Liberal Approach to the Study of Indian Politics.pdf
Liberal Approach to the Study of Indian Politics.pdfLiberal Approach to the Study of Indian Politics.pdf
Liberal Approach to the Study of Indian Politics.pdf
WaniBasim
 
Smart-Money for SMC traders good time and ICT
Smart-Money for SMC traders good time and ICTSmart-Money for SMC traders good time and ICT
Smart-Money for SMC traders good time and ICT
simonomuemu
 
DRUGS AND ITS classification slide share
DRUGS AND ITS classification slide shareDRUGS AND ITS classification slide share
DRUGS AND ITS classification slide share
taiba qazi
 
Pollock and Snow "DEIA in the Scholarly Landscape, Session One: Setting Expec...
Pollock and Snow "DEIA in the Scholarly Landscape, Session One: Setting Expec...Pollock and Snow "DEIA in the Scholarly Landscape, Session One: Setting Expec...
Pollock and Snow "DEIA in the Scholarly Landscape, Session One: Setting Expec...
National Information Standards Organization (NISO)
 
How to Build a Module in Odoo 17 Using the Scaffold Method
How to Build a Module in Odoo 17 Using the Scaffold MethodHow to Build a Module in Odoo 17 Using the Scaffold Method
How to Build a Module in Odoo 17 Using the Scaffold Method
Celine George
 
Types of Herbal Cosmetics its standardization.
Types of Herbal Cosmetics its standardization.Types of Herbal Cosmetics its standardization.
Types of Herbal Cosmetics its standardization.
Ashokrao Mane college of Pharmacy Peth-Vadgaon
 
RPMS TEMPLATE FOR SCHOOL YEAR 2023-2024 FOR TEACHER 1 TO TEACHER 3
RPMS TEMPLATE FOR SCHOOL YEAR 2023-2024 FOR TEACHER 1 TO TEACHER 3RPMS TEMPLATE FOR SCHOOL YEAR 2023-2024 FOR TEACHER 1 TO TEACHER 3
RPMS TEMPLATE FOR SCHOOL YEAR 2023-2024 FOR TEACHER 1 TO TEACHER 3
IreneSebastianRueco1
 
The Diamonds of 2023-2024 in the IGRA collection
The Diamonds of 2023-2024 in the IGRA collectionThe Diamonds of 2023-2024 in the IGRA collection
The Diamonds of 2023-2024 in the IGRA collection
Israel Genealogy Research Association
 
Executive Directors Chat Leveraging AI for Diversity, Equity, and Inclusion
Executive Directors Chat  Leveraging AI for Diversity, Equity, and InclusionExecutive Directors Chat  Leveraging AI for Diversity, Equity, and Inclusion
Executive Directors Chat Leveraging AI for Diversity, Equity, and Inclusion
TechSoup
 
A Survey of Techniques for Maximizing LLM Performance.pptx
A Survey of Techniques for Maximizing LLM Performance.pptxA Survey of Techniques for Maximizing LLM Performance.pptx
A Survey of Techniques for Maximizing LLM Performance.pptx
thanhdowork
 
How to Manage Your Lost Opportunities in Odoo 17 CRM
How to Manage Your Lost Opportunities in Odoo 17 CRMHow to Manage Your Lost Opportunities in Odoo 17 CRM
How to Manage Your Lost Opportunities in Odoo 17 CRM
Celine George
 
PCOS corelations and management through Ayurveda.
PCOS corelations and management through Ayurveda.PCOS corelations and management through Ayurveda.
PCOS corelations and management through Ayurveda.
Dr. Shivangi Singh Parihar
 
Main Java[All of the Base Concepts}.docx
Main Java[All of the Base Concepts}.docxMain Java[All of the Base Concepts}.docx
Main Java[All of the Base Concepts}.docx
adhitya5119
 
Film vocab for eal 3 students: Australia the movie
Film vocab for eal 3 students: Australia the movieFilm vocab for eal 3 students: Australia the movie
Film vocab for eal 3 students: Australia the movie
Nicholas Montgomery
 
Hindi varnamala | hindi alphabet PPT.pdf
Hindi varnamala | hindi alphabet PPT.pdfHindi varnamala | hindi alphabet PPT.pdf
Hindi varnamala | hindi alphabet PPT.pdf
Dr. Mulla Adam Ali
 

Recently uploaded (20)

বাংলাদেশ অর্থনৈতিক সমীক্ষা (Economic Review) ২০২৪ UJS App.pdf
বাংলাদেশ অর্থনৈতিক সমীক্ষা (Economic Review) ২০২৪ UJS App.pdfবাংলাদেশ অর্থনৈতিক সমীক্ষা (Economic Review) ২০২৪ UJS App.pdf
বাংলাদেশ অর্থনৈতিক সমীক্ষা (Economic Review) ২০২৪ UJS App.pdf
 
A Independência da América Espanhola LAPBOOK.pdf
A Independência da América Espanhola LAPBOOK.pdfA Independência da América Espanhola LAPBOOK.pdf
A Independência da América Espanhola LAPBOOK.pdf
 
Assessment and Planning in Educational technology.pptx
Assessment and Planning in Educational technology.pptxAssessment and Planning in Educational technology.pptx
Assessment and Planning in Educational technology.pptx
 
CACJapan - GROUP Presentation 1- Wk 4.pdf
CACJapan - GROUP Presentation 1- Wk 4.pdfCACJapan - GROUP Presentation 1- Wk 4.pdf
CACJapan - GROUP Presentation 1- Wk 4.pdf
 
Pride Month Slides 2024 David Douglas School District
Pride Month Slides 2024 David Douglas School DistrictPride Month Slides 2024 David Douglas School District
Pride Month Slides 2024 David Douglas School District
 
Liberal Approach to the Study of Indian Politics.pdf
Liberal Approach to the Study of Indian Politics.pdfLiberal Approach to the Study of Indian Politics.pdf
Liberal Approach to the Study of Indian Politics.pdf
 
Smart-Money for SMC traders good time and ICT
Smart-Money for SMC traders good time and ICTSmart-Money for SMC traders good time and ICT
Smart-Money for SMC traders good time and ICT
 
DRUGS AND ITS classification slide share
DRUGS AND ITS classification slide shareDRUGS AND ITS classification slide share
DRUGS AND ITS classification slide share
 
Pollock and Snow "DEIA in the Scholarly Landscape, Session One: Setting Expec...
Pollock and Snow "DEIA in the Scholarly Landscape, Session One: Setting Expec...Pollock and Snow "DEIA in the Scholarly Landscape, Session One: Setting Expec...
Pollock and Snow "DEIA in the Scholarly Landscape, Session One: Setting Expec...
 
How to Build a Module in Odoo 17 Using the Scaffold Method
How to Build a Module in Odoo 17 Using the Scaffold MethodHow to Build a Module in Odoo 17 Using the Scaffold Method
How to Build a Module in Odoo 17 Using the Scaffold Method
 
Types of Herbal Cosmetics its standardization.
Types of Herbal Cosmetics its standardization.Types of Herbal Cosmetics its standardization.
Types of Herbal Cosmetics its standardization.
 
RPMS TEMPLATE FOR SCHOOL YEAR 2023-2024 FOR TEACHER 1 TO TEACHER 3
RPMS TEMPLATE FOR SCHOOL YEAR 2023-2024 FOR TEACHER 1 TO TEACHER 3RPMS TEMPLATE FOR SCHOOL YEAR 2023-2024 FOR TEACHER 1 TO TEACHER 3
RPMS TEMPLATE FOR SCHOOL YEAR 2023-2024 FOR TEACHER 1 TO TEACHER 3
 
The Diamonds of 2023-2024 in the IGRA collection
The Diamonds of 2023-2024 in the IGRA collectionThe Diamonds of 2023-2024 in the IGRA collection
The Diamonds of 2023-2024 in the IGRA collection
 
Executive Directors Chat Leveraging AI for Diversity, Equity, and Inclusion
Executive Directors Chat  Leveraging AI for Diversity, Equity, and InclusionExecutive Directors Chat  Leveraging AI for Diversity, Equity, and Inclusion
Executive Directors Chat Leveraging AI for Diversity, Equity, and Inclusion
 
A Survey of Techniques for Maximizing LLM Performance.pptx
A Survey of Techniques for Maximizing LLM Performance.pptxA Survey of Techniques for Maximizing LLM Performance.pptx
A Survey of Techniques for Maximizing LLM Performance.pptx
 
How to Manage Your Lost Opportunities in Odoo 17 CRM
How to Manage Your Lost Opportunities in Odoo 17 CRMHow to Manage Your Lost Opportunities in Odoo 17 CRM
How to Manage Your Lost Opportunities in Odoo 17 CRM
 
PCOS corelations and management through Ayurveda.
PCOS corelations and management through Ayurveda.PCOS corelations and management through Ayurveda.
PCOS corelations and management through Ayurveda.
 
Main Java[All of the Base Concepts}.docx
Main Java[All of the Base Concepts}.docxMain Java[All of the Base Concepts}.docx
Main Java[All of the Base Concepts}.docx
 
Film vocab for eal 3 students: Australia the movie
Film vocab for eal 3 students: Australia the movieFilm vocab for eal 3 students: Australia the movie
Film vocab for eal 3 students: Australia the movie
 
Hindi varnamala | hindi alphabet PPT.pdf
Hindi varnamala | hindi alphabet PPT.pdfHindi varnamala | hindi alphabet PPT.pdf
Hindi varnamala | hindi alphabet PPT.pdf
 

unit 4 Elementary descrete mathematics combinatorics.pptx

  • 2. Table of contents 𝟎𝟑 04 20 07 Session 01 Fundamental Principle of Counting Multiplication Principle (Rule of AND) 04 Addition Principle (Rule of OR) Factorial Notation 𝟐𝟑 38 24 24 36 Session 02 PERMUTATION FORMULA FOR PERMUTATION Special Case Permutation when all the objects are not distinct 𝟒𝟑 44 61 Session 03 Combination String Method 𝟔𝟓 72 81 Session 04 Complementary Principle Including/Excluding Specified Objects 𝟗𝟎 91 101 102 Session 05 DIVISIBILITY MODEL Rank Circular Permutations 𝟏𝟏𝟎 112 116 Session 06 Circular Permutations Formation of Groups 𝟏𝟐𝟖 129 143 135 Session 07 All Possible Selections MODEL BASED ON DIVISORS Exponent of Prime in 𝑛! 𝟏𝟒𝟕 148 Session 08 GEOMETRY MODELS 𝟏𝟔𝟕 168 176 179 Session 09 Derangements Principle of Inclusion and Exclusion 173 Distinct Thing → Distinct Boxes Identical Things → Distinct Boxes 𝟏𝟖𝟕 Session 10 Application of Multinomial Theorem 191
  • 4. Key Takeaways Total number of ways for two events simultaneously 𝑚 × 𝑛 Event 𝐴 Different ways 𝑚 Event 𝐵 Different ways 𝑛 Fundamental Principle of Counting: Number of different ways of arranging • Counting techniques Selecting different objects Multiplication Principle (Rule of AND) Return To Top
  • 5. There are 3 ways to travel from 𝐴 to 𝐵 and 4 ways to travel from 𝐵 to 𝐶. In how many ways can a person travel from 𝐴 to 𝐶 via 𝐵. 𝐴 Return To Top 𝐵 𝐶 3 ways 4 ways Number of ways to travel from 𝐴 to 𝐵 = 3 Number of ways to travel from 𝐵 to 𝐶 = 4 Number of ways he can travel from 𝐴 to 𝐵 and 𝐵 to 𝐶 = 3 × 4 = 12 Ways
  • 6. 𝐵 𝐶 𝑥 𝑦 1 2 (𝑥, 1) (𝑥, 2) (𝑥, 3) (𝑥, 4) (𝑦, 1) (𝑦, 2) (𝑦, 3) (𝑦, 4) 𝐴 (𝑧, 1) (𝑧, 2) (𝑧, 3) (𝑧, 4) 𝑧 3 4 • The Multiplication principle can be generalised for any finite number of events. Return To Top
  • 7. Key Takeaways Addition Principle (Rule of OR): Total number of ways for an event 𝐴 or 𝐵 to occur 𝑚 + 𝑛 Event 𝐴 Different ways 𝑚 Event 𝐵 Different ways 𝑛 Return To Top
  • 8. Observe a road network in the diagram below. 𝐴 𝐵 𝐶 𝐷 In how many ways can a person travel from 𝐴 to 𝐶 via 𝐵 (or) via 𝐷. Case I:Via 𝐵 𝐴 𝐵 𝐶 3 ways 4 ways ∴ Number of ways he can travel = 3 × 4 = 12 Ways Case II:Via 𝐷 𝐴 𝐷 𝐶 2 ways 3 ways ∴ Number of ways he can travel = 2 × 3 = 6 Ways Return To Top ∴ Total number of ways = 12 + 6 = 18 Ways
  • 9. Key Takeaways Addition Principle (Rule of OR): Note:  𝐴 and 𝐵 are disjoint.  Can be extended to finite number of disjoint events. Return To Top
  • 10. In how many ways can we pair up one boy with one girl from a group of 3 boys and 5 girls ? 5 Girls 3 Boys Total possible ways = 3 × 5 = 15 15 A 3 B 5 C 8 Return To Top D
  • 11. ENTRY EXIT = 5 × 5 = 25 There are 5 doors to enter and exit the auditorium, but the same door cannot be used to enter and exit the auditorium, Then in how many ways can a student enter and exit the auditorium. Return To Top
  • 12. ENTRY EXIT = 5 × 4 = 20 There are 5 doors to enter and exit the auditorium, but the same door cannot be used to enter and exit the auditorium, Then in how many ways can a student enter and exit the auditorium. Return To Top
  • 13. 4 3 5 Customer has 4 options, Burgers Fries Beverages Deserts 2 4 × 3 × 5 × 2 = 120 Return To Top
  • 14. = 5 × 4 × 3 × 2 = 120 → → → → Return To Top
  • 15. = 12 × 11 × 10 = 1320 Twelve students compete for a race. The number of ways in which first three places can be taken is ? 2𝑛𝑑 1𝑠𝑡 3𝑟𝑑 Return To Top
  • 16. A customer forgets a 3-digit code for an Automated Teller Machine (ATM) in a bank. Find the largest possible number of unsuccessful trials necessary to obtain correct code. Return To Top 10 × 10 × 10 = 1000 = 1000 − 1 = 999 Number of unsuccessful attempts:
  • 17. Number of ways to form 3 digit numbers Number of ways of filling 3 vacant places in succession Digits :1, 2, 3, 4, 5, 6 = For 3 digit even numbers, 1 2 3 4 5 6 2 4 6 Return To Top How many 3 digit even numbers can be formed from the digits 1,2, 3, 4,5, 6 if the digits can be repeated.
  • 18. Unit’s place can be filled in 3 ways. Ten’s place can be filled in 6 ways. (Repetition is allowed) Hundred’s place also can be filled in 6 ways. (Repetition is allowed) 6 6 3 ∴ Required number of 3-digit even number = 6 × 6 × 3 = 108 Return To Top How many 3 digit even numbers can be formed from the digits 1,2, 3, 4,5, 6 if the digits can be repeated.
  • 19. i. Any of the 5 persons can leave the lift cabin in 7 different ways. Thus, by product rule, the total number of ways = 7 × 7 × 7 × 7 × 7 = 75 = 16807. 7𝑡ℎ 6𝑡ℎ 5𝑡ℎ 4𝑡ℎ 3𝑟𝑑 2𝑛𝑑 1𝑠𝑡 𝐺 ii. First person can leave the lift cabin in 7 different ways. Second person can leave the lift cabin in 6 different ways. ⋯ Thus, by product rule, the total number of ways = 7 × 6 × 5 × 4 × 3 = 2520. 5 persons entered the lift on the ground floor of an 8 −floor building (ground floor included). Suppose each of them can leave the cabin independently at any floor beginning with the first. In how many ways can each of the five persons can leave the lift i. at any of the 7 floors. ii. at different floors. Return To Top
  • 20. Key Takeaways Factorial Notation: • The product of first ‘𝑛’ natural numbers are denoted by 𝑛! 𝑛! = 1 × 2 × 3 × ⋯ × 𝑛 − 1 × 𝑛 When 𝑛 = 1,1! = 1 When 𝑛 = 2, 2! = 1 × 2 = 2 When 𝑛 = 3,3! = 1 × 2 × 3 = 6 When 𝑛 = 4,4! = 1 × 2 × 3 × 4 = 24 Return To Top
  • 21. NOTE: • We define 0! = 1 • 𝑛! = 𝑛 × 𝑛 − 1 ! = 𝑛 × 𝑛 − 1 × 𝑛 − 2 ! and so on 5! = 5 × 4! = 120 6! = 6 × 5! = 720 7! = 7 × 6! = 5040 • Factorial of a negative integer is not defined. • The value of 𝑛! ends with zero, if 𝑛 ≥ 5. Return To Top Key Takeaways
  • 22. Find the remainder when 1! + 2! + 3! + ⋯ + 100! Is divided by 15. (∵ 5!, 6!, ⋯ are divisible by 15) Return To Top Let 𝑆 = 1! + 2! + 3! + ⋯ + 100! = 1! + 2! + 3! + 4! + 5! + ⋯ + 100! = 1! + 2! + 3! + 4! + 15𝑘 𝑘 ∈ 𝐼 = 1 + 2 + 6 + 24 + 15𝑘 = 33 + 15𝑘 = 15 𝑘 + 2 + 3 ∴ Remainder is 3.
  • 23. Session 02 Permutations of distinct and alike objects Return To Top
  • 24. PERMUTATION • A permutation is an arrangement in a definite order of a number of objects taken some or all at a time. FORMULA FOR PERMUTATION • When all the objects are distinct, the number of permutations of 𝑛 distinct objects taken ′𝑟′ at a time and the objects do not repeat is denoted by 𝑛 𝑃 𝑟 or 𝑃 𝑛, 𝑟 . 𝑛𝑃𝑟 = 𝑛 × 𝑛 − 1 × 𝑛 − 2 × ⋯ × 𝑛 − 𝑟 + 1 Return To Top Key Takeaways
  • 25. 𝑛𝑃𝑟 = 𝑛 × 𝑛 − 1 × 𝑛 − 2 × ⋯ × 𝑛 − 𝑟 + 1 Proof : By definition, 𝑛𝑃𝑟 = number of permutations of 𝑟 things out of 𝑛 different things = number of ways of filling up 𝑟 vacant places with 𝑛 different objects Consider 𝑟 vacant places 1𝑠𝑡 2𝑛𝑑 3𝑟𝑑 𝑟𝑡ℎ ⋯ Return To Top 𝑛 𝑛 − 1 𝑛 − 2 𝑛 − (𝑟 − 1) So, we are filling the vacant places in succession without repetition. ∴ 𝑛𝑃𝑟 = 𝑛 × 𝑛 − 1 × 𝑛 − 2 × ⋯ × 𝑛 − 𝑟 + 1 (By Fundamental principle of counting) Key Takeaways
  • 26. • FORMULA FOR PERMUTATION 𝑟 𝑛𝑃 = 𝑛 ! 𝑛−𝑟 ! Proof : 𝑛𝑃𝑟 = 𝑛 × 𝑛 − 1 × 𝑛 − 2 × ⋯ × 𝑛 − 𝑟 + 1 𝑟 ⇒ 𝑛 𝑃 = 𝑛 × 𝑛 − 1 × 𝑛 − 2 × ⋯ × 𝑛 − 𝑟 + 1 𝑛−𝑟 ! 𝑛−𝑟 ! 𝑟 ⇒ 𝑛𝑃 = 𝑛 ! 𝑛−𝑟 ! Return To Top Key Takeaways
  • 27. 𝑛 𝑃0 = 1 (Arranging nothing) 𝑛 𝑃 𝑛 = 𝑛! (Arranging all the things) The number of permutations of 𝑛 different objects taken 𝑟 at a time,where repetition is allowed is 𝑛𝑟 . • • • Note : Key Takeaways Return To Top
  • 28. Given: 𝑛𝑃5 = 42 ⋅ 𝑛𝑃3. ⇒ 𝑛 ! = 42 ⋅ 𝑛 ! 𝑛−5 ! 𝑛−3 ! 𝑛 − 4 ⇒ 𝑛2 − 7𝑛 − 30 = 0 ⇒ 𝑛 − 10 𝑛 + 3 = 0 ⇒ 𝑛 = −3 (rejected) ⇒ 𝑛 = 10 = 42𝑛 𝑛 − 1 (𝑛 − 2) Find 𝑛 such that 𝑛𝑃5 = 42 ⋅ 𝑛𝑃3. ⇒ 𝑛 𝑛 − 1 𝑛 − 2 𝑛 − 3 ⇒ 𝑛 𝑛 − 1 𝑛 − 2 𝑛 − 3 ⇒ 𝑛 − 3 𝑛 − 4 = 42 𝑛 − 4 − 42 = 0 (∵ 𝑛 ≥ 5) Return To Top
  • 29. Prove that: 𝑖 𝑛𝑃𝑟 𝑛𝑃𝑟−1 = 𝑛 − 𝑟 + 1 Also,𝑛 𝑃 𝑟 = 𝑛 × 𝑛 − 1 × (𝑛−2)𝑃(𝑟−2) and so on 𝑖𝑖 𝑛𝑃𝑟 = 𝑛 × 𝑛−1 𝑃 𝑟−1 𝑟 𝑖 We know, 𝑛 𝑃 = 𝑛 ! 𝑛−𝑟 ! 𝑛𝑃𝑟 𝑛𝑃𝑟−1 = 𝑛−𝑟 ! 𝑛! × (𝑛−𝑟+1)! = (𝑛−𝑟+1)! = 𝑛 − 𝑟 + 1 𝑛! 𝑛−𝑟 ! 𝑛𝑃𝑟 = 𝑛 × (𝑛−1)𝑃(𝑟−1) 𝑟 𝑖𝑖 𝑛𝑃 = 𝑛−𝑟 ! 𝑛! = 𝑛 ⋅ (𝑛−1)! 𝑛−𝑟 ! Return To Top
  • 30. The value of (2 ⋅ 1𝑃0 − 3 ⋅ 2𝑃1 + 4 ⋅ 3𝑃2 − ⋯ upto 51st term) + (1! − 2! + 3! − ⋯ upto 51st term) is equal to : JEE MAINS 2020 (2 ⋅ 1𝑃0 − 3 ⋅ 2𝑃1 + 4 ⋅ 3𝑃2 − ⋯ upto 51st term) + (1! − 2! + 3! − ⋯ upto 51st term) 4 ⋅ 3𝑃2 = 4! and so on. 2 ⋅ 1𝑃0 = 2! 3 ⋅ 2𝑃1 = 3! = 2! − 3! + 4! − ⋯ − 51! + 52! + 1! − 2! + 3! − ⋯ − 50! + 51! = 1 + 52! 1 − 5! 5! A B 1 + 52! 1 + 51! D C Return To Top 1
  • 31. Three-digit numbers are to be formed by using the odd digits only. The number of such numbers which satisfy the following conditions : I. Without repetition. II. When repetition is allowed. III. When at-least one digit is repeated. Odd digits:1, 3, 5, 7, 9 I. Without repetition. 3 2! Return To Top Number of permutations without repetition = 5𝑃 = 5! = 60 II. When repetition is allowed. The number of permutations of 𝑛 different objects taken 𝑟 at a time, where repetition is allowed is 𝑛𝑟 . Number of permutations with repetition = 53 = 125.
  • 32. 125 − 60 = 65. Return To Top Odd digits:1, 3, 5, 7, 9 III. When at-least one digit is repeated. Number of permutations with repetition − Number of permutations without repetition Three-digit numbers are to be formed by using the odd digits only. The number of such numbers which satisfy the following conditions : I. Without repetition. II. When repetition is allowed. III. When at-least one digit is repeated.
  • 33. How many words, with or without meaning, can be made from the letters of the word EQUATION, using each letter exactly once if : i. 4 letters are used at a time. ii. All letters are used but first letter is a vowel. iii. All letters are used but last letter is a consonant. Given word:E Q U A T I O N 8 different letters Number of vowels = 5 Number of consonants = 3 i. 4 letters are used at a time. Number of words = 8𝑃4 = 8 × 7 × 6 × 5 = 1680. Return To Top
  • 34. _ _ _ _ _ _ _ 7! Return To Top How many words, with or without meaning, can be made from the letters of the word EQUATION, using each letter exactly once if : i. 4 letters are used at a time. ii. All letters are used but first letter is a vowel. iii. All letters are used but last letter is a consonant. O I A Number of words = 5 × 7! = 25,200 U E _ 5
  • 35. How many words, with or without meaning, can be made from the letters of the word EQUATION, using each letter exactly once if : i. 4 letters are used at a time. ii. All letters are used but first letter is a vowel. iii. All letters are used but last letter is a consonant. N T Q _ _ _ _ _ _ _ _ 3 7! Number of words = 3 × 7! = 15,120
  • 36. Key Takeaways Special Case: A C T :All the three letters are different. Return To Top • Different arrangements A C T A T C C A T C T A T A C T C A • Number of arrangements = 6 = 3! (3𝑃3)
  • 37. Key Takeaways Special Case: 3! Return To Top 2! Number of arrangements = = 3 • G E G G G E G G E E G G :Out of three, two are identical. E G G E G G • Different arrangements G E G
  • 38. Key Takeaways • Permutation when all the objects are not distinct: The number of permutations of 𝑛 objects of which 𝑝 objects are of same kind and rest are distinct = 𝑛 ! 𝑝 ! The number of permutations of 𝑛 objects where 𝑃1objects are of one • kind, 𝑃2 objects are of second kind, ⋯,𝑃𝑘 objects are of 𝑘𝑡ℎ kind and rest (if any)are all distinct = 𝑛 ! Return To Top 𝑃1!⋅𝑃2!⋯𝑃𝑘!
  • 39. Find the number of words that can be formed by using all the letters of the word : 𝑖) MESOPOTAMIA 𝑖𝑖) INDEPENDENCE Possible words = 11! 2! × 2! ×(2!) Possible words = 12! 4! × 3! ×(2!) Return To Top Total letters = 11 MM AA OO ESPTI EEEE NNN DD IPC Total letters = 12 𝑖) MESOPOTAMIA 𝑖𝑖) INDEPENDENCE
  • 40. Given word :BANANA 6! 2! ×3! = 60 6 letters ∴ Number of arrangements = ∴ Total number of words = 60 How many words can be formed using the letters of the word BANANA ? 20 A B 60 40 D C Return To Top 30
  • 41. Find the number of permutations of the letters of the word ∴ Number of permutations = 9! 4! × 2! = 7560 ∴ Number of permutations = 1 × 8! 3!×2! = 3360 ALLAHABAD. How many of them : 𝑖) start with A. 𝑖𝑖) end with L. 𝑖𝑖𝑖) start with A and end with L. Given word :A L L A H A B A D A A A A L L B D H 𝑖) start with A. A __ _ _ _ _ _ _ A A A L L B D H Return To Top
  • 42. ∴ Number of permutations = 1 × 8! = 1680 4! 𝑖𝑖𝑖) start with A and end with L. A __ _ _ _ _ _ L 3! ∴ Number of permutations = 7! = 840 Find the number of permutations of the letters of the word A A A L B D H ALLAHABAD. How many of them : 𝑖) start with A. 𝑖𝑖) end with L. 𝑖𝑖𝑖) start with A and end with L. 𝑖𝑖) end with L. _ _ _ _ _ _ _ _ L A A A A L B D H Return To Top
  • 43. Combination of objects Return To Top Session 03
  • 44. Key Takeaways • Selection of 3 objects at a time {A,B, D} {A,C, D} {B,C, D} 4 Different selections Return To Top Combination: • A combination is a selection of some or all of a number of different objects where the order of selection is immaterial. Consider four distinct objects A B {A,B, C} C D
  • 45. Key Takeaways Combination: 3 elements subset of { A, B, C, D} Return To Top • Number of combinations of 4 distinct objects taken 3 at a time = 4 {A,B, C} {A,B, D} {A,C, D} {B,C, D} In each selection, we have 3! permutations. Number of permutations = 4 × 3! = 4𝑃3
  • 46. Key Takeaways Combination: Number of selections of 3 objects out of 4 objects = 4 Number of arrangements of 3 objects taken all at a time = 3! Return To Top • • • Number of arrangements of 4 objects taken 3 at a time = 4 × 3! = 4𝑃3
  • 47. Key Takeaways Combination: taken 𝑟 at a time 𝑛 𝑃𝑟 𝑟! 𝑛 = 𝐶𝑟 In general, Return To Top Number of selections of 𝑛 objects × 𝑟! = 𝑛𝑃𝑟 Number of selections of 𝑛 objects = taken 𝑟 at a time
  • 48. Key Takeaways Combination: • The number of combinations (selections) of 𝑛 distinct objects taken 𝑟 at a time, 𝑟 is denoted by 𝑛 𝐶 or 𝐶(𝑛, 𝑟) or 𝑛 𝑟 . • 𝑛𝐶𝑟 𝑟 ! 𝑟 !(𝑛 −𝑟 )! 𝑛𝑃𝑟 𝑛! = = (𝑛 ∈ 𝑁, 𝑟 ∈ 𝑊, 0 ≤ 𝑟 ≤ 𝑛) Note 0 𝑖) 𝑛𝐶 = 𝑛! 0!𝑛 ! = 1 (Selecting nothing) 𝑛 𝑖𝑖) 𝑛𝐶 = 𝑛! 𝑛 !0! Return To Top = 1 (Selecting all the things)
  • 49. Eight teams participate in a cricket tournament. If each team plays once against each of the others, find the total number of matches ? Total :8 teams Each team plays once against each of the others DKR MSK RMI VCB DRH SR KLXI The total number of matches = 8𝐶2 = 28 DS Return To Top
  • 50. 5 boys 6 girls 5𝐶3 6𝐶4 Number of ways of selecting 3 boys = 5𝐶3 = 10 Number of ways of selecting 4 girls = 6𝐶4 = 15 ∴ Total number of selections = 10 × 15 = 150 Find the number of ways of selecting 3 boys and 4 girls from Return To Top 5 boys and 6 girls.
  • 51. How many words, with or without meaning, each of 3 vowels and 2 consonants can be formed from the letters of the word INVOLUTE ? T L Given word :INVOLUTE Vowels : I O U E Consonants : N V ∴ Number of selections of 3 vowels and 2 consonants = 4𝐶3 × 4𝐶2 For each selection, we have 5! ways Hence, number of required ways = 4𝐶3 × 4𝐶2 × 5! = 2880 Return To Top
  • 52. Find the number of 6 - digit numbers in which all the odd digits and only odd digits will appear. odd digits = 1, 3, 5, 7, 9 _ _ _ _ _ _ 6 digit number formed by using 5 odd digits ⇒ 5 odd digits are to be arranged with one digit repeating in 6 places. 1 , 3 , 5 , 7 , 9 , 1 1 , 3 , 5 , 7 , 9 , 3 1 , 3 , 5 , 7 , 9 , 5 1 , 3 , 5 , 7 , 9 , 7 1 , 3 , 5 , 7 , 9 , 9 Possible selections of digits ∴ The number of 6-digit numbers = 5𝐶1 2! × 6! = Return To Top
  • 53. Total number of cards = 52 13 cards of each type Total number of black cards = 26 T otal number of red cards J Q A 2 3 4 5 6 7 8 9 10 K A 2 3 4 5 6 7 8 9 10 K Q J A K Q J 2 3 4 5 6 7 8 9 10 A 2 3 4 5 6 7 8 9 10 K Q J Return To Top
  • 54. J Q K K Q J K Q J K Q J Total number of face cards = 12 Return To Top
  • 55. A 2 3 4 5 6 7 8 9 10 K Q J SPADE A 2 3 4 5 6 7 8 9 10 K Q J CLUB A 2 3 4 5 6 7 8 9 10 K Q J DIAMOND A 2 3 4 5 6 7 8 9 10 K Q J HEART Return To Top
  • 56. The number of ways of choosing 4 cards from a pack of 52 playing cards, such that : 𝑖) All are of the same suit is . 𝑖𝑖) Two are red and two are black cards is . 𝑖𝑖𝑖) All are face cards is . Return To Top
  • 57. Selection of suit = 4𝐶1 A 2 3 4 5 6 7 8 9 10 K Q J A 2 3 4 5 6 7 8 9 10 K Q J A 2 3 4 5 6 7 8 9 10 K Q J A 2 3 4 5 6 7 8 9 10 K Q J Selection of 4 cards from same suit = 13𝐶4 𝑖) All are of the same suit = 4𝐶1 × 13𝐶4 = 2860 Return To Top
  • 58. The number of ways of choosing 4 cards from a pack of 52 playing cards, such that : Return To Top 𝑖) All are of the same suit is . 𝑖𝑖) Two are red and two are black cards is . 𝑖𝑖𝑖) All are face cards is . Selection of two black cards = 26𝐶2 Selection of two red cards = 26𝐶2 𝑖𝑖) Two are red and two are black cards = 26𝐶2 × 26𝐶2 = 105625 𝑖𝑖𝑖) All are face cards = 13𝐶4 = 495
  • 59. Selection of one heart = 13𝐶1 Selection of one spade = 13𝐶1 One is heart and other is spade = 13𝐶1 × 13𝐶1 = 169 Find the number of ways of choosing 2 cards from a pack of 52 playing cards, such that one is heart and other one is spade. D A B C Return To Top 13𝐶1 × 13𝐶1 26𝐶1 × 26𝐶1 52𝐶2 13𝐶2
  • 60. Given word :RANDOM Vowels : A O Find the number of permutations of the letters of the words RANDOM such that vowels come together. Consonants : R N D M Consider as a single unit AO R N D M 5 ∴ Number of arrangements of 5 units = 5! AO OA In each arrangement, two vowels can be arranged in 2! Ways. ∴ Total number of arrangements = 5! × 2! = 240 Return To Top
  • 61. The number of permutations of 𝑛 distinct things, taken all at A time when 𝑚 specified things always come together. 𝑚! 𝑛 − 𝑚 + 1 ! Return To Top String Method
  • 62. 6 women The number of ways in which we can choose a committee from 4 men and 6 women so that the committee includes at least two men and exactly twice as many women as men is . 4 men Committee: At least 2 men and twice as many women as men. Case 𝑖 : Two men and Four women Number of selections = 4𝐶2 × 6𝐶4 Case 𝑖𝑖 : Three men and six women Number of selections = 4𝐶3 × 6𝐶6 ∴ Total number of ways = 4𝐶2 × 6𝐶4 + 4𝐶3 × 6𝐶6 = 94. Return To Top
  • 63. There are 𝑚 men and two women participating in a chess tournament. Each participant plays two games with every other participant. If the number of games played by the men between themselves exceeds the number of games played between the men and women by 84, then the value of 𝑚 is . Given: Number of men = 𝑚, Number of women = 2, Number of games = 𝑚𝐶2 × 2 vs vs Number of games = 𝑚𝐶1 × 2𝐶1 × 2 Return To Top
  • 64. 2 × 𝑚𝐶2 = 2 × 𝑚𝐶1 × 2𝐶1 + 84 ⇒ 2 × 𝑚 𝑚−1 2! = 4 𝑚 + 84 ⇒ 𝑚2 − 𝑚 = 4 𝑚 + 84 ⇒ 𝑚2 − 5 𝑚 − 84 = 0 ⇒ 𝑚 − 12 𝑚 + 7 = 0 𝑚 = −7 neglected 𝑚 = 12 There are 𝑚 men and two women participating in a chess tournament. Each participant plays two games with every other participant. If the number of games played by the men between themselves exceeds the number of games played between the men and women by 84, then the value of 𝑚 is . Return To Top
  • 65. Permutations I ncluding /Excluding specified Objects Return To Top Session 04
  • 66. Given: Total number of questions = 12 Number of questions to be attempted = 8 CASE PART (I) PART (II) Number of ways Total : 5 Total : 7 𝑖 3 5 5𝐶3 × 7 𝐶5 𝑖𝑖 4 4 5𝐶4 × 7 𝐶4 𝑖𝑖𝑖 5 3 5𝐶5 × 7 𝐶3 Return To Top In an examination, a question paper consists of 12 questions divided into two parts ,part (𝑖) and part (𝑖𝑖), containing 5 and 7 questions, respectively. A student is required to attempt 8 questions in all, selecting at least 3 from each part. In how many ways can a student select the questions?
  • 67. ∴ Total number of ways in which the student can select = 5𝐶3 × 7𝐶5 + 5𝐶4 × 7𝐶4 + 5𝐶5 × 7𝐶3 = 210 + 175 + 35 = 420 Return To Top In an examination, a question paper consists of 12 questions divided into two parts ,part (𝑖) and part (𝑖𝑖), containing 5 and 7 questions, respectively. A student is required to attempt 8 questions in all, selecting at least 3 from each part. In how many ways can a student select the questions?
  • 68. The number of ways in which 5 boys and 3 girls can be arranged in a row such that : i) All the boys are together . ii) All the girls are not together . 3 girls 5 boys i)All the boys are together Consider as a single unit. 5 boys sitting together, taken as a single unit. Return To Top
  • 69. i)All the boys are together 4! ways Return To Top 5! ways ∴ Total number of arrangements = 4! × 5! = 2880 ii)All the girls are not together ∴ Total number of arrangements in which all the girls are not together = (Number of ways of arranging 5 boys and 3 girls) − (Number of ways of arranging in which all the girls are together)
  • 70. 3 girls sitting together, taken as a single unit ii)All the girls are not together (Number of ways of arranging 5 boys and 3 girls) = 5 + 3 ! And (Number of ways of arranging in which all the girls are together) 1 + 5 = 6 units Return To Top
  • 71. 3! ways ∴ Total number of arrangements = 3! × 6! ∴ Total number of arrangements in which all the girls are not together = 5 + 3 ! − 6! × 3! = 8! − 6! × 3! = 6! 8 × 7 − 6 = 36000 Return To Top ii)All the girls are not together 6! ways
  • 72. Complementary Principle Note: In the previous problem, we used complementary principle. Number of favorable ways = Total number of ways − Number of unfavourable ways Return To Top
  • 73. 4 boys 4 girls i)No two girls are together 5 4 So, 4 boys can be arranged in 4! ways and 4 girls can be arranged in five places in 𝐶 × 4! ways ∴ Total number of ways = 4! × 5𝐶4 × 4! = 2880 The number of ways in which 4 boys and 4 girls can be arranged in a row such that: Return To Top i) No two girls are together . ii)Boys and girls sit alternatively.
  • 74. 4 girls can be arranged in 4! ways. 4 boys can be arranged in 4! ways. ∴ Total number of ways = 4! × 4! Hence,the total number of ways = 4! × 4! + 4! × 4! = 1152 The number of ways in which 4 boys and 4 girls can be arranged in a row such that: i) No two girls are together . ii)Boys and girls sit alternatively. ii)Boys and girls sit alternatively Case 1 : 4 boys can be arranged in 4! ways. 4 girls can be arranged in 4! ways ∴ Total number of ways = 4! × 4! Case 2: Return To Top
  • 75. And consonants can be arranged in 7! 2! ∴ Total number of required arrangements = 2 × 5! × 7! 2! = 604800 Find the number of arrangements of the letters of the word PERMUTATIONS such that : Return To Top i) All the vowels as one unit and all the consonants as one unit. ii) There are always 4 letters between 𝑃 and 𝑆. i) All the vowels as one unit and all the consonants as one unit. Vowels :E U A I O Consonants: P R M T T N S There are two units, which can be arranged in = 2! ways In each arrangement in vowels can be arranged in = 5! ways
  • 76. Remaining 10 places can be filled with the letters 10! 2! 2! ∴ Total number of ways = 7 × 2 × 10! = 25401600 ERMUTATION 𝑃 𝑆 𝑆 𝑃 2 cases 1+1+1+1+1+1+1 = 7 Find the number of arrangements of the letters of the word PERMUTATIONS such that : Return To Top i) All the vowels as one unit and all the consonants as one unit. ii) There are always 4 letters between 𝑃 and 𝑆. ii)There are always 4 letters between 𝑃 and 𝑆.
  • 77. ∴ Total number of ways = 3! × 4! = 144 5𝑡ℎ 6𝑡ℎ 7𝑡ℎ 1𝑠𝑡 2𝑛𝑑 3𝑟𝑑 4𝑡ℎ 3! ways 4! ways The number of permutations of the letters of the word HEXAGON such that Return To Top i)relative positions of the vowels and consonants are not changed. ii)Order of the vowels is not changed. Given word :HEXAGON i) relative positions of the vowels and consonants are not changed. Vowels : E A O Consonants : H X G N
  • 78. 4 7𝐶4 × 4! Given word :HEXAGON Vowels : E A O Consonants : H X G N ii)Order of the vowels is not changed Let us arrange the consonants in 4 out of 7 places. This can be done in Now remaining three places can be filled with vowels in only one way. (∵ order of the vowels not to be changed) ∴ Total number of ways = 7𝐶4 × 4! × 1 = 840 The number of permutations of the letters of the word HEXAGON such that Return To Top i)relative positions of the vowels and consonants are not changed. ii)Order of the vowels is not changed.
  • 79. Note : The number of permutations of 𝑛 distinct things, in which the order of 𝑟 things 𝑟 ! Return To Top is not to be considered is 𝑛 ! = 𝑛 𝑃 𝑛− 𝑟
  • 80. Including/Excluding Specified Objects The number of combination of 𝑛 distinct things taken 𝑟 at a time, such that 𝑖) 𝑝 particular things are always included is 𝑖𝑖) 𝑝 particular things are always excluded is 𝑛−𝑝 𝐶 𝑟−𝑝 𝑛−𝑝 𝐶𝑟 𝑖𝑖𝑖) 𝑝 particular things are included and 𝑞 particular things are excluded is 𝑛−𝑝−𝑞 𝐶 𝑟−𝑝 Return To Top Key Takeaways
  • 81. The number of ways in which a team of 11 players can be selected from 22 players such that i) 2 particular players are always included is ii) 4 particular players are always excluded is iii)2 particular players are included and 4 particular players are excluded is i) 2 particular players are always included is Given :𝑛 = 22, 𝑟 = 11 ∴ 𝑛−𝑝 𝐶 𝑟−𝑝 = 20𝐶9 ii) 4 particular players are always excluded is ∴ 𝑛−𝑝 𝐶𝑟 = 18𝐶11 Return To Top
  • 82. ∴ 𝑛−𝑝−𝑞 𝐶 𝑟−𝑝 = 16𝐶9 Return To Top The number of ways in which a team of 11 players can be selected from 22 players such that i) 2 particular players are always included is ii) 4 particular players are always excluded is iii)2 particular players are included and 4 particular players are excluded is iii) 2 particular players are always included and 4 particular players are excluded is
  • 83. ∴ Number of required subsets = Given, 𝐴 = 𝐸, 𝑄, 𝑈, 𝐴, 𝑇, 𝐼, 𝑂, 𝑁 ∴ 𝑛 = 8, 𝑟 = 5 Condition 𝐴, 𝑇 are always included 8−2 𝐶 5−2 = 6𝐶3 = 20 If set 𝐴 = 𝐸, 𝑄, 𝑈,𝐴, 𝑇, 𝐼, 𝑂, 𝑁 then the number of 5 elements. Subsets of 𝐴 which always include 𝐴, 𝑇 is Return To Top
  • 84. A guard of 12 men is formed from a group of 𝑛 soldiers. It is found that two particular soldiers 𝐴, 𝐵 is 3 times as often together on guard as three particular soldiers 𝐶, 𝐷, 𝐸. Then the value of 𝑛 is . Number of soldiers = 𝑛, 𝑟 = 12 Number of ways in which 𝐴, 𝐵 are included is 3 times that of 𝐶, 𝐷, 𝐸 included on guard. Return To Top
  • 85. C Return To Top E B A D Number of ways in which 𝐴, 𝐵 are included is 3 times that of 𝐶, 𝐷, 𝐸 included on guard. Number of ways in which A and B are included = 𝑛 −2𝐶12−2 Number of ways in which C, D and E are included = 𝑛−3𝐶12−3 A guard of 12 men is formed from a group of 𝑛 soldiers. It is found that two particular soldiers 𝐴, 𝐵 is 3 times as often together on guard as three particular soldiers 𝐶, 𝐷, 𝐸. Then the value of 𝑛 is .
  • 86. 𝑛−2 𝐶 12−2 ⇒ = 3 × 𝑛−3 𝐶 12−3 ⇒ 𝑛−2 𝐶10 = 3 × 𝑛−3 𝐶9 ⇒ 𝑛−2 ! 10! 𝑛−12 ! = 3 × 𝑛−3 ! 9! 𝑛−12 ! ⇒ 𝑛−2 = 3 10 ⇒ 𝑛 = 32 Return To Top A guard of 12 men is formed from a group of 𝑛 soldiers. It is found that two particular soldiers 𝐴, 𝐵 is 3 times as often together on guard as three particular soldiers 𝐶, 𝐷, 𝐸. Then the value of 𝑛 is .
  • 87. 5 girls 7 boys Consider a class of 5 girls and 7 boys. The number of different teams consisting of 2 girls and 3 boys that can be formed from this class, if there are two specific boys 𝐴 and 𝐵 who refuse to be the member of the same team, is 7𝐶3 Number of ways of selecting 2 girls = 5𝐶2 Number of ways of selecting 3 boys = Team consists of 2 girls and 3 boys Total number of ways to select 2 girls and 3 boys for the team = 5𝐶2 × 7𝐶3 = 10 × 35 = 350 Return To Top
  • 88. 5 girls 7 boys selecting 1 boy = 5𝐶1 Consider a class of 5 girls and 7 boys. The number of different teams consisting of 2 girls and 3 boys that can be formed from this class, if there are two specific boys 𝐴 and 𝐵 who refuse to be the member of the same team, is Number of ways of selecting 2 girls = 5𝐶2 Number of ways of A B If the two boys are included, only one boy has to be chosen from 5 boys ∴ Number of ways the team can be formed if both the boys are included = 5𝐶2 × 5𝐶1 = 10 × 5 = 50 ∴ Required number of selections = (Total no. of ways) − (No. of ways the team can be formed if both the boys are included) = 350 − 50 = 300 Return To Top
  • 89. JEE MAINS 2019 A B C D 350 500 200 300 Consider a class of 5 girls and 7 boys. The number of different teams consisting of 2 girls and 3 boys that can be formed from this class, if there are two specific boys 𝐴 and 𝐵 who refuse to be the member of the same team, is Return To Top
  • 90. Session 05 Rank of a word and Circular Permutations Return To Top
  • 91. 0 Multiple of 7 504,3367,5719 DIVISIBILITY MODEL: • A number is divisible by 3: If the sum of its digits is divisible by 3. • A number is divisible by 4: If it’s last 2 digits are divisible by 4. • A number is divisible by 6: If it is divisible by 2 and 3. • A number is divisible by 7: if [2 × units-place digits − (Number formed by other digits)] Example: Return To Top
  • 92. DIVISIBILITY MODEL: • A number is divisible by 8: If it’s last 3 digits are divisible by 8. • A number is divisible by 9: If the sum of its digits is divisible by 9. • A number is divisible by 11: if (sum of the digits in the odd places) − (sum of digits in even places) is divisible by 11. Example: 209,3564 Return To Top
  • 93. Number of 5 digited numbers using the digits 0, 1, 2, 3,4 with repetition,which are divisible by 4, is: Given digits:0, 1, 2, 3, 4 A number is divisible by 4 if its last two digits are divisible by 4. Last two digits can be : 00 04 12 20 24 32 40 44 8 ways 4 × 5 × 5 × 8 ∴ Total number of ways = 4 × 5 × 5 × 8 = 800. 800 A 1000 B 400 C 600 Return To Top D
  • 94. 216 A 600 B 240 C 3125 D Given digits:0, 1, 2, 3, 4 and 5 A number is divisible by 3 if the sum of its digit is a multiple of 3. 3 3 + 9 = 12 9 The sum of given six digits = 0 + 1 + 2 + 3 + 4 + 5 = 15 15 ⇒ Possible sum of five digits such that the number formed is divisible by 3 = 12 or 15 Return To Top A five-digit number divisible by 3 is to be formed using the numbers 0, 1, 2, 3, 4 and 5 without repetitions. The total number of ways this can be done is:
  • 95. Case 𝑖 : Digits are 0, 1, 2, 4, 5 (Sum of digits = 12) Number of ways = 5𝑃5 − (Number of ways in which 0 comes in first place) = 5! − 4! = 120 − 24 = 96 Case 𝑖𝑖 : Digits are 1, 2, 3, 4, 5 (Sum of digits = 15) Number of ways = 5𝑃5 = 5! = 120 ∴ Total number of ways = 96 + 120 = 216 A five-digit number divisible by 3 is to be formed using the numbers 0, 1, 2, 3, 4 and 5 without repetitions. The total number of ways this can be done is: Return To Top
  • 96. If the letters of the word MOTHER are permutated and all the words so formed (with or without meaning) be listed as in a dictionary, then the position of the word ‘MOTHER’ is . Given word :MOTHER Alphabetic order of letters : E, H, M, O, R,T From the given letters, the word MOTHER comes after the words : 1) Starting with E 2) Starting with H 3) Starting with M 𝐴) Starting with ME 𝐵) Starting with MH 𝑎) Starting with MOE 𝑏) Starting with MOH 𝐶) Starting with MO 𝑐) Starting with MOR 𝑑) Starting with MOT 𝑖) Starting with MOTE Return To Top Next Word :MOTHER [JEE MAIN 2020 ]
  • 97. Starting with E E 5! = 120 Starting with H H 5! = 120 Starting with ME M 4! = 24 E Starting with MH M 4! = 24 H Starting with MOE M O E Starting with MOH M O H Starting with MOR M = 6 3! = 6 3! = 6 3! O R Starting with MOTE M 2! = 2 O T E Return To Top
  • 98. Starting with E E = 120 Starting with H H = 120 Starting with ME M = 24 E Starting with MH M = 24 H Starting with MOE M = 6 O E Starting with MOH M = 6 O H Starting with MOR M = 6 O R Starting with MOTE M = 2 O E Next word M O T H E R T Return To Top
  • 99. ∴ Position of the word MOTHER = 309𝑡ℎ Return To Top Words Number of words Sum Starting with E 120 − Starting with H 120 240 Starting with ME 24 264 Starting with MH 24 288 Starting with MOE 6 294 Starting with MOH 6 300 Starting with MOR 6 306 Starting with MOTE 2 308
  • 100. Given word :AGAIN Alphabetic order of letters :A, A, G,I,N Starting with A A Starting with G G 4! 2! I = 12 4! 2! If all the permutations of the letters of the word AGAIN are arranged in the order as in a dictionary, then find the 49𝑡ℎ word. Words Number of words Sum Starting with 𝐴 24 Starting with 𝐺 12 36 Starting with 𝐼 12 48 49 = 12 4! = 24 The next word is N A Return To Top A G I Starting with I
  • 101. Rank: The position of a word when all permutations of that word are written in alphabetical order is called rank. Example: Let the word be CAT. All possible permutation of the word CAT: CAT, CTA, ATC, TCA, ACT, TAC. Arranging them in alphabetical order CAT: ACT, ATC, CAT, CTA, TAC, TCA CAT is the 3𝑟𝑑 in the above list, so the rank of the word CAT is 3. Return To Top
  • 102. Key Takeaways Circular Permutations The number of circular permutations of 𝑛 distinct objects is 𝑛 − 1 ! 𝑎1 𝑎2 𝑎3 𝑎𝑛 𝑎𝑛−1 Return To Top
  • 103. Key Takeaways Proof : Let 𝑥 be the number of circular permutations of 𝑛 distinct objects. For each circular arrangement, the number of linear arrangements = 𝑛 ∴ The number of linear arrangements of 𝑛 distinct objects = 𝑛 × Number of circular arrangements of 𝑛 distinct things 𝑛! = 𝑛 × 𝑥 𝑛 ∴ 𝑥 = 𝑛! = 𝑛 − 1 ! ∴ The number of circular permutations of 𝑛 distinct objects is 𝑛 − 1 ! Return To Top
  • 104. Key Takeaways Circular Permutations The number of circular permutations of 𝑛 distinct objects is 𝑛 − 1 ! 𝑎1 𝑎2 𝑎3 𝑎𝑛 𝑎𝑛−1 Difference between linear and circular arrangement : Linear → Recognized starting place Circular → No starting and ending place Return To Top Note: If positional places are marked, then circular arrangements can be treated as linear arrangements.
  • 105. i) There is no restriction. 9 children can be arranged in 9 − 1 ! = 8! ways The number of ways in which 5 boys and 4 girls can be seated at a round table, if Return To Top i)there is no restriction. ii) all the girls are together . iii) all the girls are not together . iv)no two girls are not together .
  • 106. ii)All the girls are together. Consider 4 girls as one unit. ∴ Total 6 units 6 units can be arranged in 6 − 1 ! = 5! ways. 4 girls can be arranged themselves in 4! ways. Required number of ways = 5! × 4! iii)All the girls are not together. Required number of ways = Total number of ways − Number of ways in which all girls are together = 8! − 5! × 4! Return To Top
  • 107. iv)No two girls are not together. We can arrange 5 boys in 4! ways. There will be 5 gaps to arrange 4 girls= 4! × 5C4 Total number of ways = 4! × 5C4 × 4! Return To Top
  • 108. Out of 10 people, 5 are to be selected around a round table and the remaining to be seated in a row. The number of arrangements is . Total :10 people 5! ways Number of linear arrangements = 5! Number of circular permutations of 𝑛 distinct things, taken 𝑟 at a time is 𝑛𝐶𝑟 . 𝑟 − 1 ! Return To Top
  • 109. Required number of circular permutations = 10𝐶5 . 5 − 1 ! = 10𝐶5 . 4! Required number of linear arrangements = 5! Total number of arrangements = 10𝐶5 . 4! ⋅ 5! 1 Return To Top 5 = ⋅ 10! = 2 × 9! Out of 10 people, 5 are to be selected around a round table and the remaining to be seated in a row. The number of arrangements is .
  • 110. Session 06 Division and Distribution of objects Return To Top
  • 111. Number of ways = 2 8 − 1 ! 1 2 = × 7! = 2520 The number of ways in which a garland can be made by using 8 Return To Top different flowers is .
  • 112. Key Takeaways 2 directions are taken as not different is 1 𝑛 − 1 ! . Circular Permutations : Note : Number of circular permutations of 𝑛 distinct things, if clockwise and anticlockwise Return To Top Example :Circular arrangement of flowers in a Garland, Beads in a Necklace.
  • 113. 2 𝑟 when clockwise and anticlockwise directions are same is 1 .𝑛𝐶 . 𝑟 − 1 ! 2 𝑟 Required number of ways = 1 .𝑛𝐶 . 𝑟 − 1 ! 1 2 18 = × 𝐶12 × 12 − 1 ! 1 Return To Top 18 = × 𝐶12 × 11! 2 How many necklaces of 12 beads each can be made from 18 beads of various colors? . Number of circular permutations of 𝑛 distinct things, taken 𝑟 at a time
  • 114. Note : Number of circular permutations of 𝑛 distinct things, taken 𝑟 at a time is 𝑛𝐶𝑟 . 𝑟 − 1 !. In the above case, if clockwise and anticlockwise directions are taken as not different, 2 𝑟 then the number of circular permutations is 1 .𝑛𝐶 . 𝑟 − 1 !. Return To Top
  • 115. How many different necklaces be made from 10 different beads such that three particular beads always come together. Necklaces be made from 10 different beads = 2 8−1 ! × 3! = 7! × 3! 2 8 different beads including 3 beads together as 1 bead 9! 2 A 7! 2 B 9!×3! 2 C 7!×3! Return To Top 2 D
  • 116. Key Takeaways Formation of Groups: Let us consider 4 people Return To Top
  • 117. Number of ways = 4𝐶1 × 3𝐶3 = 4! 1!3! = 4 Key Takeaways Formation of Groups: 𝐼) Let us divide them into two unequal groups of sizes 1 and 3 Return To Top
  • 118. and 𝑛 is 𝑚+𝑛 ! 𝑚! 𝑛 ! • Number of ways of dividing 𝑚 + 𝑛 + 𝑝 (𝑚 ≠ 𝑛 ≠ 𝑝) things into three unequal groups of size 𝑚, 𝑛, 𝑝 is 𝑚+𝑛+𝑝 ! 𝑚! 𝑛! 𝑝! Key Takeaways Results: • Number of ways of dividing 𝑚 + 𝑛 (𝑚 ≠ 𝑛) things into two unequal groups of size 𝑚 𝑘 𝑛 is • Number of ways of dividing 𝑛1 + 𝑛2 + ⋯ + 𝑛𝑘 into 𝑘 unequal groups of size 𝑛1, 𝑛2, … , (𝑛1+ 𝑛2 +⋯+ 𝑛𝑘)! 𝑛1! 𝑛2! … 𝑛𝑘! General Results: Return To Top
  • 119. Key Takeaways 𝐼𝐼) Let us divide them into two equal groups 2! 4𝐶2×2𝐶2 Number of ways = = 4! 2!(2!∙2!) = 3 Return To Top
  • 120. • Number of ways of dividing 𝑘𝑛 things into 𝑘 equal groups of size 𝑛 each is 𝑘𝑛 ! 𝑘! 𝑛! 𝑘 Key Takeaways Results: • Number of ways of dividing 2𝑛 things into two equal groups of size 𝑛 each is 2𝑛 ! 2!(𝑛!∙𝑛!) • Number of ways of dividing 3𝑛 things into three equal groups of size 𝑛 each is 3𝑛 ! 3! 𝑛!∙𝑛!∙𝑛! General Results: Return To Top
  • 121. 𝑖) 10 2 3 5 𝑖𝑖) Number of ways = 10! 2!3!5! Number of ways = 10! 2! 5! 2 𝑖𝑖𝑖) 𝑖𝑣) Number of ways = 10! 2!2! 4! 2 10! 3! 3! 31! Consider division of 10 distinct things into groups, in different cases. 10 2 4 4 10 5 5 10 Return To Top 3 3 3 Number of ways = 1
  • 122. 10 2 2 2 2 2 10 1 4 10 4 6 10 2 3 Number of ways = 10! 5! 2! 5 2 3 Number of ways = 10! 1!2!3!4! Number of ways = 10! 4!6! 2 3 Number of ways = 10! 2! 2! 2. 2! 3! 2 Return To Top 𝑣) 𝑣𝑖) 𝑣𝑖𝑖) 𝑣𝑖𝑖𝑖)
  • 123. 𝑖) Number of ways = 52! 4! 13! 4 Number of ways = 52! 4! 13! 4 × 4! = 52! 13! 4 Four equal sets 𝑖𝑖) Distributing equally among four people Four equal sets can be distributed among 4 people in 4! ways 52 Return To Top 13 13 13 13 The number of ways in which a pack of 52 cards: 𝑖) can be divided into four equal sets is . 𝑖𝑖) can be distributed among four people equally is .
  • 124. Number of ways = 5! 1!2! 2! 2 = 15 Number of ways = 5! 2! 1! 23! = 10 Case 1 Total number of ways = 15 + 10 = 25 5 1 2 2 Case 2 5 1 Return To Top 1 3 Find the number of ways in which 5 books can be 𝑖) tied up in 3 bundles. 𝑖𝑖) distributed among 3 students such that each student get at-least one book. 𝑖) 5 books → 3 bundles
  • 125. 1!2! 2! 2 Number of ways = 5! = 15 ⋅ 3! = 90 (Distribution) Total number of ways = 90 + 60 = 150 𝑖𝑖) 5 books → 3 students Case 1 5 1 2 2 5! 2! 1! 23! = 10 ⋅ 3! = 60 Number of ways = (Distribution) Case 2 5 1 Return To Top 1 3
  • 126. 𝑃1 𝑃2 𝑃3 𝑃4 A boat is manned by 8 men, 4 on each side. In how many ways can 8 men be arranged if two particular men can only row on left side and another one particular man can only row on right side? 8 men → 4 on each side 𝑃5 𝑃6 𝑃7 𝑃8 Return To Top
  • 127. 5! Remaining 5 people will be divided into two groups of size 2 and 3 in ways 2!3! Here we have to arrange the crew on both the sides, which can be done in 4! ways 5! 2!3! Total number of arrangements = × 4! × 4! = 5760 𝑃8 𝑃7 𝑃6 𝑃5 𝑃1 𝑃2 𝑃3 𝑃4 Return To Top
  • 128. Session 07 Selections from distinct and Identical objects Return To Top
  • 129. Key Takeaways All Possible Selections: Case 1: • The total number of selections out of 𝑛 distinct things taken any number of things (including nothing)is 2𝑛 • Total number of selections = 𝑛𝐶0 + 𝑛𝐶1 + ⋯ + 𝑛𝐶𝑛 = 2𝑛 • The total number of ways of selecting at least one thing from 𝑛 distinct things = 2𝑛 − 1 • Total number of selections = 𝑛𝐶1 + 𝑛𝐶2 + ⋯ + 𝑛𝐶𝑛 = 2𝑛 − 𝑛𝐶0 = 2𝑛 − 1 Return To Top
  • 130. = 10𝐶1 + 10𝐶2 + ⋯ + 10𝐶10 = 210 − 10𝐶0 Number of ways = 210 − 1 = 1023 There are 10 lamps in a room, each can be switched on independently. The number of ways in which the room can be illuminated is . Return To Top
  • 131. Key Takeaways All Possible Selections: Case 2: • The total number of selections out of 𝑛 similar things taken any number of things (including nothing)is 𝑛 + 1. • The total number of ways of selecting at-least one thing from 𝑛 similar things = 𝑛. If there are three similar letters,say A,A and A,then Total number of selections = {no A, one A, two A’s, three A’s}= 4 ways Case 3: • If there are 𝑝 similar things of one type, 𝑞 similar things of second type and 𝑟 similar things of third type, then the number of ways of selecting atleast one thing is 𝑝 + 1 𝑞 + 1 𝑟 + 1 − 1 Return To Top
  • 132. Number of ways of selecting at least one fruit = 119 4 + 1 = 5 + 1 3 + 1 −1 Return To Top Find the number of ways of selecting at least one fruit from 5 mangoes, 4 apples and 3 bananas.
  • 133. Key Takeaways Case 4: • If there are 𝑝, 𝑞, 𝑟 similar things of different kinds and 𝑛 distinct things, then the number of ways of selecting atleast one thing is 𝑝 + 1 𝑞 + 1 𝑟 + 1 2𝑛 − 1 Return To Top
  • 134. 2 Case II:2 alike of 1 kind and 2 alike of other kind: 4! 2!2! Number of ways = 3𝐶 × = 18 Case III:2 alike and 2 distinct 1 2 2! Number of ways = 3𝐶 × 6𝐶 × 4! = 540 Case IV:4 distinct Number of ways = 7𝐶4 × 4! = 840 ∴ Number of 4 letter words = 24 + 18 + 540 + 840 = 1422 Four letters words: Case I:3 alike and 1 distinct. 1 3! Number of ways = 1 × 6𝐶 × 4! = 24 We can get 4 four letter words, if we permutates the letters in each of the above cases. Number of 4 letter words with or without meaning that can be formed using letters of the word INEFFECTIVE is: Return To Top
  • 135. Key Takeaways MODEL BASED ON DIVISORS: • Divisors of 12 are: 1 2 3 4 6 12 • Number of divisors = 12 Sum of divisors = 28 Let us explain these two with ‘combinations’ 12 = 22 ∙ 31 (prime factorization) Return To Top
  • 136. 2 = 21 ∙ 30 3 = 20 ∙ 31 4 = 22 ∙ 30 6 = 21 ∙ 31 12 = 22 ∙ 31 The number of divisors = (Number of ways of selecting two 2’s)× (Number of ways of selecting one 3) = 2 + 1 1 + 1 = 6 • So, if 12 = 2𝑥 ∙ 3𝑦 then number of divisors = 𝑥 + 1 𝑦 + 1 Key Takeaways MODEL BASED ON DIVISORS: 12 = 22 ∙ 31 (prime factorization) 1 = 20 ∙ 30 Return To Top
  • 137. Sum of divisors = 20 ∙ 30 + 21 ∙ 30 + 20 ∙ 31 +22 ∙ 30 +21 ∙ 31 + 22 ∙ 31 = (20 + 21 + 22)(30 + 31) = (7)(4) = 28 So, if 12 = 2𝑥 ∙ 3𝑦 ,then Sum of the divisors = (20 + 21 + 22 + ⋯ 2𝑥)(30 + 31 + 32 + ⋯ 3𝑦) G.P . Return To Top G.P. Key Takeaways
  • 138. Key Takeaways 1 𝑝𝑥1+1 −1 𝑝1−1 2 𝑝𝑥2+1 −1 𝑝2−1 ⋯ • → Sum of all the divisors = MODEL BASED ON DIVISORS: Let 𝑁 be a positive integer and 𝑁 = 𝑝𝑥1 ⋅ 𝑝𝑥2 ⋅ 𝑝𝑥3 ⋯ 1 2 3 Where 𝑝1, 𝑝2, 𝑝3 ⋯ are distinct primes and 𝑥1, 𝑥2, 𝑥3 ⋯ ∈ 𝑁 • → Number of divisors of 𝑁 = 𝑥1 + 1 𝑥2 + 1 𝑥3 + 1 0 1 1 1 1 2 • → Sum of all the divisors = 𝑝 + 𝑝 + 𝑝 + ⋯ + 𝑝1 𝑥1 2 Return To Top 0 2 1 2 2 2 𝑥2 𝑝 + 𝑝 + 𝑝 + ⋯ 𝑝 ⋯
  • 139. 𝑥1 + 1 𝑥2 + 1 ⋯ , if 𝑁 is not a perfect square MODEL BASED ON DIVISORS: Let 𝑁 be a positive integer and 𝑁 = 𝑝𝑥1 ⋅ 𝑝𝑥2 ⋅ 𝑝𝑥3 ⋯ 1 2 3 Where 𝑝1, 𝑝2, 𝑝3 ⋯ are distinct primes and 𝑥1, 𝑥2, 𝑥3 ⋯ ∈ 𝑁 Number of ways in which 𝑁 can be resolved as a product of two factors is equal to • 1 2 • 1 2 1 𝑥 + 1 2 Return To Top 𝑥 + 1 ⋯ + 1 , if 𝑁 is a perfect square
  • 140. iii. Sum of divisors = 22−1 34−1 53−1 2−1 3−1 5−1 = 3720 iv. Sum of proper divisor = 3720 − 1 + 1350 = 2369 Return To Top Consider the number 1350. Find i. Number of divisors ii. Number of proper divisors iii. Sum of divisors iv. Sum of proper divisors Given: 1350 = 21 ∙ 33 ∙ 52 i. Number of divisors = 1 + 1 3 + 1 2 + 1 = 24 ii. Number of proper divisor = 24 − 2 = 22 ( Except 1 and 1350 )
  • 141. In how many ways the number 8100 can be written as product of two coprime factors? Given: 8100 = 22 ∙ 34 ⋅ 52 Possible ways to express 8100 as product of two coprime factors are, → 1 × 22 ∙ 34 ∙ 52 → 22 × 34 ∙ 52 → 34 × (22 ∙ 52) → 52 × 22 ∙ 34 ∴ Number of ways = 4 Return To Top
  • 142. • If ′𝑛′ is the number of different prime factors of 𝑁, then the number of ways in which 𝑁 can be resolved as product of two coprime factors is equal to 2𝑛−1. MODEL BASED ON DIVISORS: Return To Top
  • 143. Prime Factorization of 𝑛! Exponent of Prime in 𝑛! • Let 𝑛! = 2𝑒1 ⋅ 3𝑒2 ⋅ 5𝑒3 ⋅ 7𝑒4 ⋯ Example: 3! = 21 ⋅ 31 4! = 23 ⋅ 31 5! = 23 ⋅ 31 ⋅ 51 100! =? Return To Top Key Takeaways
  • 144. 1 𝑒 = 𝑛 𝑝1 + 𝑛 𝑝12 + 𝑛 𝑝13 + ⋯ 2 𝑒 = 𝑛 𝑝2 + 𝑛 𝑝22 + 𝑛 𝑝23 + ⋯ Exponent of 𝑝2 in 𝑛! Is given as Exponent of Prime in 𝑛! • Let 𝑝 be a prime number and 𝑛 be any positive integer such that : 𝑛! = 𝑝1 𝑒1 ⋅ 𝑝2 𝑒2 ⋅ 𝑝3 𝑒3 ⋯ Exponent of 𝑝1 in 𝑛! Is given as Key Takeaways . denotes greatest integer Return To Top
  • 145. 200! = 2𝑒1 ⋅ 3𝑒2 ⋅ 5𝑒3 ⋯ 1 𝑒 = 200 2 + 200 22 + 200 23 + 200 24 + 200 25 + 200 26 + 200 27 + ⋯ = 100 + 50 + 25 + 12 + 6 + 3 + 1 + 0 = 197 3 𝑒 = 200 5 + 200 52 + 200 53 + 200 54 + ⋯ = 40 + 8 + 1 + 0 = 49 200! = 2197 ⋅ 3𝑒2 ⋅ 549 ⋯ = 2148 ⋅ 249 ⋅ 3𝑒2 ⋅ 549 ⋯ = 2148 ⋅ 3𝑒2 ⋅ 1049 ⋯ Number of zeros in 200! = 49. Find the number of zeros in 200! Return To Top
  • 146. Find the number of zeros in 200! 200 5 = 40 40 5 = 8 8 5 = 1 1 5 Return To Top = 0 Number of zeros in 200! = 40 + 8 + 1 + 0 = 49 Alternative solution: Number of zeros in 200! directly equals to multiples of 10 in 200! Among the prime factors 2 and 5 the highest power of 5 in 200! will be less than the highest power of 2 in 200! So, the highest power of 10 in 200! will be equal to highest power of 5 in 200!
  • 147. Session 08 Applications of selections in geometry Return To Top
  • 148. HAND SHAKE PROBLEM Key Takeaways GEOMETRY MODELS • If there are 𝑛 points in a plane (no three are collinear) then by joining them we can obtain: i. 𝑛 𝐶2 straight lines ii.𝑛𝐶3 triangles 𝑃1 𝑃2 𝑃3 𝑃4 Return To Top
  • 149. Key Takeaways GEOMETRY MODELS • If there are 𝑛 points in a plane of which 𝑚 points are collinear, then by joining them, we can obtain i. 𝑛 𝐶2 − 𝑚𝐶2 + 1 straight lines ii.𝑛𝐶3 − 𝑚𝐶3 triangles Return To Top
  • 150. • 𝑛𝐶2 − 𝑛 = 𝑛 𝑛−3 2 Key Takeaways GEOMETRY MODELS The number of diagonals of a polygon of 𝑛 sides (𝑛 vertices) is Return To Top
  • 151. 𝑚𝐶2 × 𝑛𝐶2 Key Takeaways GEOMETRY MODELS If a set of 𝑚 parallel lines are intersected by another set of 𝑛 parallel lines, then the number of parallelograms, we can obtain is • Return To Top
  • 152. 𝑛𝐶2 i. 𝑛 straight lines in a plane is ii. 𝑛 circles in a plane is 2 × 𝑛 𝐶2 Key Takeaways GEOMETRY MODELS The maximum number of points of intersection of Return To Top
  • 153. There are 12 points in a place of which 7 are collinear. By joining them,we can have : Return To Top i. Number of straight lines ii. Number of triangles iii.Number of circles iv.Number of pentagons i. Number of straight lines = 12𝐶2 − 7𝐶2 + 1 = 46 ii. Number of triangles = 12𝐶3 − 7𝐶3 = 185 iii. Number of circles = 12𝐶3 − 7𝐶3 = 185 iv. Number of pentagons = 12𝐶5 − 7𝐶5 = 771
  • 154. The maximum number of points of intersection of : Return To Top i)20 lines in a plane is . ii) 10 circles in a plane is . iii)20 lines and 10 circles in a plane is . i)20 line in a plane The maximum number of points of intersection of 𝑛 straight lines in a plane is 𝑛 𝐶2 ⇒ Maximum number of points of intersection of 20 straight lines = 20𝐶2 = 190 The maximum number of points of intersection of 𝑛 circles in a plane is 2 × 𝑛𝐶2. ⇒ Maximum number of points of intersection of 10 circles = 2 × 10𝐶2 = 2 × 45 = 90
  • 155. Case 𝑖 = 2 × 20𝐶1 × 20𝐶1 = 400 Case 𝑖𝑖 = 2 × 10𝐶2 = 90 Case 𝑖𝑖𝑖 Return To Top = 2 × 10𝐶2 = 90 Total = 400 + 90 + 190 = 680
  • 156. A convex polygon has 44 diagonals. Find the number of its sides. ⋮ ⋮ ⋮ 44 diagonals Let 𝑛 be the number of slides ⇒ 𝑛 points (vertices) The number of diagonals of a polygon of 𝑛 sides (𝑛 vertices) is 𝑛𝐶2 − 𝑛. ⋮ 𝑉1 𝑉2 𝑉𝑛 𝑛𝐶2 − 𝑛 = 2 𝑛 (𝑛−1) − 𝑛 = 44 ⇒ 𝑛 (𝑛−3) = 44 2 ⇒ 𝑛 𝑛 − 3 = 88 ⇒ 𝑛2 − 3𝑛 − 88 = 0 𝑉3 ⇒ 𝑛 − 11 𝑛 + 8 = 0 Return To Top ⇒ 𝑛 = 11 (∵ 𝑛 = −8 is not possible)
  • 157. 𝐵 𝐶 Triangle can be formed by 𝐴 𝐴𝐵 → 3 points 𝐵𝐶 → 4 points 𝐴𝐶 → 5 points Selecting one point each on one side Selecting two points on one side and third point on any other side OR In a triangle 𝐴𝐵𝐶, the sides 𝐴𝐵, 𝐵𝐶 and 𝐴𝐶 have 3, 4 and 5 points respectively on them.The number of triangles that can be constructed using these points as vertices is . Return To Top
  • 158. 𝐵 𝐶 Case (i) Selecting one point each on = 3𝐶1 × 4𝐶1 × 5𝐶1 = 60 one side Case (ii) Selecting two points on one side and third point on any other side = 3𝐶2 × 9𝐶1 + 4𝐶2 × 8𝐶1 + 5𝐶2 × 7𝐶1 = 27 + 48 + 70 = 145 ∴ Number of triangles = 60 + 145 = 205 Return To Top
  • 159. 𝑛 points Given : 𝑇𝑛 +1 − 𝑇𝑛 = 21 Number of triangles = 𝑇𝑛 = 𝑛 𝐶3, 𝑇𝑛 +1 = 𝑛 +1𝐶3 𝑇𝑛+1 − 𝑇𝑛 = 21 𝑛𝐶2 + 𝑛𝐶3 = 𝑛+1𝐶3 ⇒ 𝑛+1𝐶3 − 𝑛𝐶3 = 21 ⇒ 𝑛𝐶2 = 21 ⇒ 𝑛 𝑛 − 1 = 42 ⇒ 𝑛 = 7 Let 𝑇𝑛 denotes the number of triangles which can be formed by joining the 𝑛 points which lie on a circle. If 𝑇𝑛+1 − 𝑇𝑛= 21, then the value of 𝑛 is . Return To Top
  • 160. Given : A regular polygon of 10 sides two vertices are consecutive Number of ways in which no = Number of ways of selecting any 3 vertices. − Number of ways of selecting 3 consecutive vertices. − Number of ways of selecting 3 vertices such that two are consecutive. A regular polygon of 10 sides is constructed. In how many ways can 3 vertices be selected so that no two vertices are consecutive ? Return To Top
  • 161. Let the vertices be :𝑉1, 𝑉2, 𝑉3, ⋯ , 𝑉10 Number of ways of selecting any 3 vertices = 10𝐶3 𝑉1 𝑉2 𝑉3, 𝑉2 𝑉3 𝑉4 , ⋯ , 𝑉10 𝑉1 𝑉2 𝑉2 𝑉1 𝑉3 𝑖) Starting with 𝑉 𝑉 = 6 𝑉1 𝑉2 𝑉4 𝑉7 𝑉8 𝑉9 6 ways 𝑉6 𝑉5 Return To Top 𝑉4 𝑉5 𝑉6 𝑉7 𝑉8 𝑉9 𝑉10 1 2 Similarly,we have starting with 𝑉2 𝑉3, 𝑉3 𝑉4, 𝑉4𝑉5, ⋯ , 𝑉10 𝑉1 Number of ways of selecting 3 consecutive vertices = 10 Number of ways of selecting 3 consecutive vertices such that two are consecutive = 10 × 6 = 60
  • 162. Number of ways of selecting any 3 vertices = 10𝐶3 Number of ways of selecting 3 consecutive vertices = 10 Number of ways of selecting 3 vertices such that two are consecutive = 60 Number of ways in which no two vertices are consecutive = 10𝐶3 − 10 − 60 = 120 − 70 = 50 𝑉 Return To Top 2 𝑉1 𝑉3 𝑉4 𝑉5 𝑉6 𝑉7 𝑉8 𝑉9 𝑉10
  • 163. How many squares are there in a chess board? A chessboard has 64 small squares. There are many more different sized squares on the chessboard. Return To Top 1. No.of 8 × 8 𝑠𝑞. = 1 2. No.of 7 × 7 𝑠𝑞. = 4 3. No.of 6 × 6 𝑠𝑞. = 9 4. No.of 5 × 5 𝑠𝑞. = 16 5. No.of 4 × 4 𝑠𝑞. = 25 6. No.of 3 × 3 𝑠𝑞. = 36 7. No.of 2 × 2 𝑠𝑞. = 49 8. No.of 1 × 1 𝑠𝑞. = 64 ∴ Total number of squares on chessboard ⇒ 64 + 49 + 36 + 25 + 16 + 9 + 4 + 1 = 204
  • 164. Given word : ROORKEE Let A ≡ {words begin with R } B ≡ {words end with E } A’ ∩ B’ ≡ {words neither begin with R nor end with E } ROORKEE 𝑛 𝑈 = 7! 2! . 2! . 2! 𝑛 𝐴 = 6! 2! . 2! R Find the number of words that can be made by permutating all the letters of the word ROORKEE, which neither begins with R nor end with E. Return To Top
  • 165. Let A ≡ {words begin with R } B ≡ {words end with E } A’ ∩ B’ ≡ {words neither begin with R nor end with E } ROORKEE 𝑛 𝑈 = 7! 2! . 2! . 2! 𝑛 𝐴 = 6! 2! . 2! R E 𝑛 𝐵 = 6! 2! . 2! 𝑛 𝐴 ∩ 𝐵 5! 2! R E = Return To Top
  • 166. ROORKEE 𝑛 𝐴 = 6! 2! . 2! 𝑛 𝐵 = 6! 2! . 2! 𝑛 𝐴 ∩ 𝐵 = 5! 2! 𝑛 𝐴′ ∩ 𝐵′ = 𝑛 𝑈 = 𝑛 𝑈 − 𝑛 𝐴 𝖴 𝐵 − 𝑛 𝐴 − 𝑛 𝐵 + 𝑛 𝐴 ∩ 𝐵 = 7! 6! − − 6! + 5! 2! .2! .2! 2! . 2! 2! . 2! 2! 𝑛 𝑈 = 7! Return To Top 2! . 2! . 2! = 630 − 180 − 180 + 60 = 330
  • 167. Session 09 Distribution of objects to distinct boxes Return To Top
  • 168. Key Takeaways 𝑛 𝐷 = 𝑛! 1 − 1 + 1 − 1 + ⋯ + −1 𝑛 1 1! 2! 3! 𝑛 ! where 𝑛 ≥ 2 Derangements If 𝑛 things are arranged in a row, then the number of rearrangements such that none of them occupy their original positions are called Dearangements. The number of Derangements of 𝑛 distinct things can be denoted by 𝐷𝑛 . Return To Top
  • 169. Given word : BAG All permutations = 3! = 6 ways = ABG AGB BAG BGA GAB GBA Number of derangements = 𝐷3 = 2 Find the number of derangements of the letters of the word BAG. 𝑛 𝐷 = 𝑛! 1 − 1 + 1 − 1 + ⋯ + −1 𝑛 1 1! 2! 3! 𝑛 ! where 𝑛 ≥ 2 3 1! 2! 3! 𝐷 = 3! 1 − 1 + 1 − 1 = 2 Using formula: Return To Top
  • 170. Six letters with numbers 1,2,3,4,5,6written on them are put in six envelopes with numbers 1,2,3,4,5,6written on them too. 𝑖) No letter is in its correct envelope is 𝑖𝑖) At least one letter is in its correct envelope is 𝑖𝑖𝑖) At most two letters are not in their correct envelopes is 𝑖𝑣) Exactly three letters are not in their correct envelopes is Given: . . Lette r 1 . Lette r 2 Lette r 3 Lette r 4 Lette r 5 . . . . . . Return To Top . . . . . . . . . Lette r 6 1 2 3 4 5 6
  • 171. 𝑖) No letter is in its correct envelope is . . Lette r 1 . Lette r 2 Lette r 3 Lette r 4 Lette r 5 . . . . . . Return To Top . . . . . . . . . Lette r 6 1 2 3 4 5 6 Number of ways = 𝐷6= 265 𝑖𝑖) At least one letter is in its correct envelope is Number of ways =Total number of permutations − No letter is in its correct envelope = 6! − 265 = 455
  • 172. 𝑖𝑖𝑖) At most two letters are not in their correct envelopes is + + All letters are in its correct envelope One letter isn’t in its correct envelope Two letters are not in their correct envelope = 1 + 0 + 6𝐶2 × 𝐷2 = 16 𝑖𝑣) Exactly three letters are not in their correct envelopes is = 6𝐶3 × 𝐷3 = 40 Return To Top
  • 173. Principle of Inclusion and Exclusion 𝑛 𝐴 𝖴 𝐵 = + - 𝑛 𝐴 𝑛 𝐵 𝑛 𝐴 ∩ 𝐵 include exclude 𝑛 𝐴 𝖴 𝐵 = 𝑛 𝐴 + 𝑛 𝐵 − 𝑛 𝐴 ∩ 𝐵 Return To Top
  • 174. 𝐴 𝐵 𝐶 𝑛 (𝐵 ∩ 𝐶) 𝑛 (𝐴 ∩ 𝐵 ∩ 𝐶) 𝑛 𝐴 𝖴 𝐵 𝖴 𝐶 = 𝑛 (𝐴) + 𝑛 (𝐵) + 𝑛 (𝐶) − 𝑛 (𝐴 ∩ 𝐵) − 𝑛 (𝐴 ∩ 𝐶) − + include include exclude Principle of Inclusion and Exclusion Return To Top
  • 175. In general, 𝑛 𝐴1 𝖴 𝐴2 𝖴 𝐴3 𝖴 ⋯ 𝖴 𝐴𝑛 = ෍ 𝑛 𝐴𝑖 − ෍ 𝑛 𝐴𝑖 ∩ 𝐴𝑗 + ෍ 𝑛 𝐴𝑖 ∩ 𝐴𝑗 ∩ 𝐴𝑘 − ⋯ + −1 𝑛 𝑛 𝐴1 ∩ 𝐴2 ∩ 𝐴3 ∩ ⋯ ∩ 𝐴𝑛 ⇒ 𝑛 𝐴1′ 𝖴 𝐴2′ 𝖴 𝐴3′ 𝖴 ⋯ 𝖴 𝐴𝑛′ = 𝑛 𝑈 − 𝑛 𝐴1 𝖴 𝐴2 𝖴 𝐴3 𝖴 ⋯ 𝖴 𝐴𝑛 Return To Top Principle of Inclusion and Exclusion
  • 176. Key Takeaways Distinct Thing → Distinct Boxes: The number of ways of distributing 𝑛 distinct things in 𝑟 distinct boxes such that each box is filled with Case 1 0 or more things (i.e. empty boxes are allowed) = 𝑟𝑛 Case 2 Atleast one thing (i.e. empty boxes are not allowed) = 𝑟𝑛 − 𝑟 𝐶1 𝑟 − 1 𝑛 + 𝑟 𝐶2 𝑟 − 2 𝑛 − ⋯ + −1 𝑟−1 ∙ 𝑟𝐶𝑟−1 1 𝑛 Return To Top
  • 177. In how many ways 5 different balls can be distributed into 3 boxes so that no box remains empty. = 35 − 3𝐶1 3 − 1 5 + 3𝐶2 3 − 2 5 − 3𝐶3 3 − 3 5 = 150 The required number of ways Atleast one ball (i.e. empty boxes are not allowed) −1 𝑟−1 ∙ 𝑟𝐶𝑟−1 1 𝑛 Return To Top = 𝑟𝑛 − 𝑟𝐶1 𝑟 − 1 𝑛 + 𝑟𝐶2 𝑟 − 2 𝑛 − ⋯ + where 𝑛 = 5, 𝑟 = 3
  • 178. 𝑖𝑖) Number of functions from 𝐴 to 𝐵 such that every element in 𝐵 has atleast one pre-image If 𝐴 = 𝑎1, 𝑎2, 𝑎3, 𝑎4, 𝑎5 and 𝐵 = 𝑏1, 𝑏2, 𝑏3 then find 𝑖) Number of functions from 𝐴 to 𝐵 = 𝑟𝑛 = 35 = 243 𝑩 𝑨 𝒃𝟏 𝒃𝟐 𝒃𝟑 = 𝑟𝑛 − 𝑟𝐶1 𝑟 − 1 𝑛 + 𝑟𝐶2 𝑟 − 2 𝑛 − ⋯ + = 35 − 3𝐶1 3 − 1 5 + 3𝐶2 3 − 2 5 = 150 −1 𝑟−1 ∙ 𝑟𝐶𝑟−1 1 𝑛 𝒂𝟏 𝒂𝟐 𝒂𝟑 𝒂𝟒 𝒂𝟓 If 𝐴 = 𝑎1,𝑎2, 𝑎3, 𝑎4, 𝑎5 and 𝐵 = 𝑏1,𝑏2, 𝑏3 then find Return To Top 𝑖) Number of functions from 𝐴 to 𝐵 𝑖𝑖) Number of functions from 𝐴 to 𝐵 such that every element in 𝐵 has atleast one pre-image
  • 179. Key Takeaways Proof: Let 𝑛 identical objects put on floor 1 2 3 4 𝑛 − 1 ⋯ ⋯ 𝑛 Identical Things → Distinct Boxes: The number of ways of distributing 𝑛 distinct things in 𝑟 distinct boxes such that each box is filled with Return To Top Case 1 0 or more things (i.e. empty boxes are allowed) = 𝑛+𝑟−1𝐶𝑟−1
  • 180. 𝑟 − 1 identical partitions 𝑛 identical objects 𝑟 − 1 identical partitions 𝑛! 𝑟−1 ! 𝑛+𝑟−1 ! = 𝑛+𝑟−1𝐶𝑟 −1 Proof (Cont.⋯): To form 𝑟 −groups we need 1 2 3 4 ⋯ ⋯ 𝑛 − 1 Return To Top 𝑛 Key Takeaways
  • 181. 𝑛−1𝐶𝑟−1 0 or more things (i.e.empty boxes are allowed) = 𝑛+𝑟−1𝐶𝑟−1 Now where 𝑛 − 𝑟 objects to be distributed in 𝑟 boxes. = 𝑛−𝑟+𝑟−1𝐶𝑟−1 = 𝑛−1𝐶𝑟−1 Key Takeaways Return To Top Identical Things → Distinct Boxes: The number of ways of distributing 𝑛 distinct things in 𝑟 distinct boxes such that each box is filled with Case 2 Atleast one thing (i.e. empty boxes are not allowed) = Proof: Let us give each box 1object to satisfy atleast one condition.
  • 182. Key Takeaways Identical Things → Distinct Boxes: For the equation 𝑥1 + 𝑥2 + 𝑥3 + ⋯ + 𝑥𝑟 = 𝑛 Where 𝑛 is 𝑛 identical units of 1. Return To Top 𝑖) Number of non-negative integral solution 𝑛 +𝑟 −1𝐶𝑟 −1 𝑖𝑖) Number of positive integral solution 𝑛−1𝐶𝑟−1
  • 183. In how many ways 5 identical balls can be distributed into 3 distinct boxes so that no box remains empty. Return To Top Given: 𝑛 = 5, 𝑟 = 3 The required number of ways = 𝑛−1𝐶𝑟−1 = 4𝐶2 = 6
  • 184. For the equation 𝑎 + 𝑏 + 𝑐 + 𝑑 = 12. Return To Top 𝑖) Number of non-negative integral solutions are 𝑖𝑖) Number of positive integral solutions are 𝑖) Number of non-negative integral solutions are Given: 𝑛 = 12, 𝑟 = 4 𝑛+𝑟−1𝐶𝑟−1 = 12+4−1𝐶4−1 = 15𝐶3 = 455 𝑖𝑖) Number of positive integral solutions are 𝑛−1𝐶𝑟−1 = 12−1𝐶4−1 = 11𝐶3 = 165
  • 185. In how many ways 10 apples, 5 mangoes, 4 oranges can be distributed among 4 persons, when each person may receive any number of fruits. (Fruits of each type are identical) Return To Top Given: Number of persons = 4 Let 𝑥1, 𝑥2, 𝑥3, 𝑥4 be number of apples receive by 𝑃1,𝑃2,𝑃3,𝑃4respectively ∴ 𝑥1 + 𝑥2 + 𝑥3 + 𝑥4 = 10 10+4−1𝐶4−1 = 13𝐶3 Let 𝑦1, 𝑦2, 𝑦3, 𝑦4 be number of mangoes receive by 𝑃1,𝑃2,𝑃3,𝑃4respectively ∴ 𝑦1 + 𝑦2 + 𝑦3 + 𝑦4 = 5 5+4−1𝐶4−1 = 8𝐶3
  • 186. Let 𝑧1, 𝑧2, 𝑧3, 𝑧4 be number of oranges receive by 𝑃1,𝑃2,𝑃3,𝑃4 respectively ∴ 𝑧1 + 𝑧2 + 𝑧3 + 𝑧4 = 4 4+4−1𝐶4−1 = 7𝐶3 The required number of ways = 13𝐶3 × 8𝐶3 × 7𝐶3 = 560560 Return To Top In how many ways 10 apples, 5 mangoes, 4 oranges can be distributed among 4 persons, when each person may receive any number of fruits. (Fruits of each type are identical)
  • 187. Multinomial theorem and its applications Return To Top Session 10
  • 188. 16 3 3 3 3 4 𝑎 𝑏 𝑐 𝑑 𝑎 + 𝑏 + 𝑐 + 𝑑 = 4 4+4−1𝐶4−1 = 7𝐶3 = 35 Given: Remaining 4 to be distributed among 4 Persons so that any one receive any amount,which is equal to the number of non-negative integral solutions of In how many ways can Rs. 16 be divided among 4 persons such that none of them gets less than Rs. 3. Return To Top
  • 189. Find the number of non-negative integral solutions of 𝑎 + 𝑏 + 𝑐 + 𝑑 ≤ 20. Return To Top Given: 𝑎 + 𝑏 + 𝑐 + 𝑑 ≤ 20 ⋯ (𝑖) Let 𝑒 ≥ 0 be a dummy variable such that 𝑎 + 𝑏 + 𝑐 + 𝑑 + 𝑒 = 20 ⋯ (𝑖𝑖) Now, number of non-negative integral solutions of (𝑖) = number of non-negative integral solutions of (𝑖𝑖) = 𝑛+𝑟−1𝐶𝑟−1 𝑛 = 20, 𝑟 = 5 = 20+5−1𝐶5−1
  • 190. 𝑥 + 𝑦 + 𝑧 = 20 𝑢 − 4 + 𝑣 + 1 + 𝑧 = 20 𝑢 + 𝑣 + 𝑧 = 23⋯ (𝑖𝑖) where 𝑢 ≥ 0, 𝑣 ≥ 0 and 𝑧 ≥ 0 Given: 𝑥 + 𝑦 + 𝑧 = 20 ⋯ (𝑖) 𝑥 ≥ −4 ⇒ 𝑥 + 4 ≥ 0 𝑦 ≥ 1 ⇒ 𝑦 − 1 ≥ 0 Let,𝑥 + 4 = 𝑢 and 𝑦 − 1 = 𝑣 Number of required solutions of (𝑖) = Number of non- negative integral solutions of (𝑖𝑖) = 23+3−1𝐶3−1 = 25𝐶2 = 300 Find the number of integral solutions of 𝑥 + 𝑦 + 𝑧 = 20 where 𝑥 ≥ −4, 𝑦 ≥ 1, ≥ 𝑧 ≥ 0. Return To Top
  • 191. Key Takeaways Application of Multinomial Theorem In this theorem we try to write all possible outcomes in powers of a random variable 𝑥 and then calculate coefficient of required outcomes. Return To Top
  • 192. Find the number of ways of distributing 10 identical apples among 3 children without any restrictions Let 𝑎, 𝑏, 𝑐 be number of apples distributed to three children 𝑎 + 𝑏 + 𝑐 = 10; 𝑎 ≥ 0, 𝑏 ≥ 0. 𝑐 ≥ 0 Possible outcomes = 𝑎 𝑥0 + 𝑥1 + 𝑥2 + ⋯ 𝑏 𝑥0 + 𝑥1 + 𝑥2 + ⋯ 𝑐 𝑥0 + 𝑥1 + 𝑥2 + ⋯ Number of solution will be = Coefficient of 𝑥10 in 𝑥0 + 𝑥1 + 𝑥2 + ⋯ 𝑥0 + 𝑥1 + 𝑥2 + ⋯ 𝑥0 + 𝑥1 + 𝑥2 + ⋯ Infinite G.P . Return To Top
  • 193. Coefficient of 𝑥𝑟 in 1 − 𝑥 −𝑚 is 𝑚+𝑟−1𝐶𝑟 Coefficient of 𝑥10 in 1 1 1 1−𝑥 1−𝑥 1−𝑥 = Coefficient of 𝑥10 in 1 − 𝑥 −3 𝑎 𝑆∞ = 1−𝑟 = 66 = 10+3−1𝐶10 = 12𝐶10 𝑚 = 3, 𝑟 = 10 Return To Top
  • 194. Find the number of non-negative integral solutions of 𝑥1 + 𝑥2 + 𝑥3 + 4𝑥4 = 20. Given: 𝑥1+𝑥2 + 𝑥3 + 4𝑥4 = 20 Number of non-negative integral solutions = the coefficient of 𝑥20 in the product 1 + 𝑥 + 𝑥2 + ⋯ 3 1 + 𝑥4 + 𝑥8 + ⋯ 𝑚+1 𝐶2 𝑥2 − 𝑚+2 𝐶3 𝑥3 + ⋯ = coefficient of 𝑥20 in 1 − 𝑥 −3 1 − 𝑥4 −1 1 + 𝑥 −𝑚 = 1 − 𝑚𝐶1 𝑥 + = Coefficient of 𝑥20 in (1 + 3𝐶1𝑥 + 4𝐶2𝑥2 + ⋯ + 𝑟+2𝐶𝑟𝑥𝑟) 1 + 𝑥4 + 𝑥8 + ⋯ = 1 + 6𝐶4 + 10𝐶8 + 14𝐶12 + 18𝐶16 + 22𝐶20 = 536 Return To Top
  • 195. Given : 15days → to visit for 4 days, No two visits are consecutive. Let 𝑉1, 𝑉2, 𝑉3, 𝑉4 be the 4 visiting days An engineer is required to visit a factory for exactly 4 days during the first 15 days of every month and it is mandatory that no two visits take place on consecutive days. Then the number of all possible ways in which such visits to the factory can be made by the engineer during 1-15June 2021 is . Return To Top JEE (ADVANCED)2020
  • 196. = 8+5−1𝐶5−1 = 12𝐶4 = 495 𝑎 𝑏 𝑐 𝑑 𝑉1 𝑉2 𝑉3 𝑉4 Return To Top 𝑒 where, 𝑎, 𝑏, 𝑐, 𝑑, 𝑒 be the remaining days So, 𝑎 + 𝑏 + 𝑐 + 𝑑 + 𝑒 = 11 and 𝑎 ≥ 0, 𝑏 ≥ 1, 𝑐 ≥ 1, 𝑑 ≥ 1, 𝑒 ≥ 0 Put 𝑏 = 1 + 𝑥, 𝑐 = 1 + 𝑦, 𝑑 = 1 + 𝑡. then, 𝑎 + 𝑥 + 𝑦 + 𝑡 + 𝑒 = 8 and 𝑎 ≥ 0, 𝑥 ≥ 0, 𝑦 ≥ 0, 𝑡 ≥ 0, 𝑒 ≥ 0 Number of non-negative integral solution = 𝑛+𝑟−1𝐶𝑟−1 Let 𝑉1, 𝑉2, 𝑉3, 𝑉4 be the 4 visiting days Consider,
  • 197. Key Takeaways Result : If 𝑑1, 𝑑2, ⋯ , 𝑑𝑛 are the given non-zero digits, then the sum of all 𝑛 digit numbers (without repetition) is equal to 𝑛 − 1 ! (𝑑1 + 𝑑2 + ⋯ + 𝑑𝑛 ) (100 + 101 + ⋯ + 10𝑛 −1) Return To Top
  • 198. Find the sum of all four-digit numbers formed by using the digits 2, 3, 4,5 without repetition. _ Given digits:2, 3, 4, 5 _ _ _ _ Numbers having 2 in unit’s place = 3! 5 4 3 2 3! Similarly,Numbers having 3, 4, 5 in units place = 3! _ _ _ _ 5 4 3 2 3! Sum of all digits in unit’s place = 3! 2 + 3 + 4 + 5 Return To Top Number of 4-digit numbers = 4! = 24 _ _ _ Click to add tex4 t!
  • 199. _ _ _ _ 2 Sum of all digits in unit’s place = 3! Number having 2 in ten’s place= 3! 2 + 3 + 4 + 5 3 4 Similarly,Numbers having 3, 4, 5 in 5 ten’s place = 3! _ _ _ _ Sum of digits in ten’s place =3! 2 + 3 + 4 + 5 × 10 Sum of all digits = Sum of all digits of unit’s ,ten’s, hundred’s and thousand’s places. Hence, the sum of all 4 digited number = 3! 2 + 3 + 4 + 5 1111 Return To Top = 3! 2 + 3 + 4 + 5 100 + 101 + 102 + 103 = 6 × 14 × 1111 = 93324